You are on page 1of 97

FLOW THROUGH PIPES

Definition of flow through pipes


A pipe is a closed conduit carrying a fluid under pressure. Fluid motion in a pipe
is subjected to a certain resistance. Such a resistance is assumed to be due to
Friction. In reality this is mainly due to the viscous property of the fluid.
Reynolds Number (Re)
It is defined as the ratio of Inertia force of a flowing fluid and the Viscous force.
Re=(Inertia force/Viscous force) =( V D/ )
Classification of pipe flow:
Based on the values of Reynolds number (Re), flow is classified as Follows:
Laminar flow or Viscous Flow
In such a flow the viscous forces are more predominent compared to inertia Forces.
Stream lines are practically parallel to each other or flow takes place In the form of
telescopic tubes. This type of flow occurs when Reynolds number Re< 2000. In laminar
flow velocity increases gradually from zero at the boundary to Maximum at the center.
Laminar flow is regular and smooth and velocity at any point practically remains constant
in magnitude & direction. Therefore, the flow is also known as stream Line flow.
There will be no exchange of fluid particles from one layer to another. Thus there
will be no momentum transmission from one layer to another. Ex: Flow of thick oil
in narrow tubes, flow of Ground Water, Flow of Blood in blood vessels.
Transition flow:
In such a type of flow the stream lines get disturbed a little. This type of flow
occurs when 2000< Re < 4000.
Laminar flow Transition flow Turbulent flow
Water
Dye
Glass tube
Hydraulic Grade Line & Energy Grade Line
A Line joining the peizometric heads at various points in a flow is known as Hydraulic
Grade Line (HGL)
Energy Grade Line (EGL)
It is a line joining the elevation of total energy of a flow measured above a datum, i.e.
EGL Line lies above HGL by an amount V
2
/2g.
Losses in Pipe Flow
Losses in pipe flow can be two types viz:-
a)Major Loss
b)Minor Loss
a)Major Loss: As the name itself indicates, this is the largest of the losses in a pipe. This
loss occurs due to friction only. Hence, it is known as head loss due to friction (h
f
)
b)Minor Loss: Minor losses in a pipe occurs due to change in magnitude or direction of
flow.
Minor losses are classified as (i) Entry Loss, (ii) Exit loss, (iii) Sudden expansion
loss (iv) Sudden contraction loss (v) Losses due to bends & pipe fittings.
Head Loss due to Friction
Consider the flow through a straight horizontal pipe of diameter D, Length L, between two sections (1) & (2)
as shown. Let P
1
& P
2
be the pressures at these sections. To is the shear stress acting along the pipe
boundary.
(1)
(1)
(2)
(2)
L
D
p
1
p
2
Flow (V)
From II Law of Newton
Force = Mass x accn. But acceleration = 0, as there is no change in velocity, however the reason
that pipe diameter is uniform or same throughout.
Applying Bernoullis equation between (1) & (2) with the centre line of the pipe as
datum & considering head loss due to friction h
f
,.
Substituting eq (2) in eq.(1)
From Experiments, Darcy Found that
( )
( ) ) 1 (
4
4
4 4
. .
0
0
2 1
0
2
2 1
0
2
2
2
1


+

D
L
P P or
DL
D
P P
DL x
D
P
D
P e i
forces



f
h
g
V p
Z
g
V p
Z + + + + +
2 2
2
2 2
2
2
1 1
1

2 1
Z Z

2 1
V V
Pipe is horizontal
Pipe diameter is
same throughout
) 2 (
2 1

f
h
P P

) 4 (
8
2
0
V
f

f=Darcys friction factor (property of the pipe materials Mass density of the liquid.
V = velocity
Equations (3) & (4)
But,
from Continuity equation
& (5) & (6) are known as DARCY WEISBACH Equation
Pipes in Series or Compound Pipe
D
1
, D
2
, D
3
, D
4
are diameters.
L
1
, L
2
,L
3
, L
4
are lengths of a number of Pipes connected in series
(hf)
1
, (hf)
2
, (hf)
3 &
(hf)
4
are the head loss due to friction for each pipe.
The total head loss due to friction h
f
for the entire pipe system is given by
or
D
V Lf
h
f

8
4
2

) 5 (
2
2

,
_

gD
fLV
h
g
f

) 6 (
8
5 2
2

,
_


D gh
fLQ
h
f
D2
D3
D4 D1
Q
L2 L4
L1
4 3 2 1
hf hf hf hf h
f
+ + +
5
4
2
2
4
5
3
2
2
3
5
2
2
2
2
5
1
2
2
1
8 8 8 8
D g
Q fL
D g
Q fL
D g
Q fL
D g
Q fL
h
f

+ + +
D
1
, D
2
and D
3
are the pipe diameters. Length of each pipe is same, that is, L
1
=L
2
=L
3
For pipes in parallel hf1=hf2=hf3 i.e
Equivalent pipe
In practice adopting pipes in series may not be feasible due to the fact that they may be of
unistandard size (ie. May not be comemercially available) and they experience other
minor losses. Hence, the entire system will be replaced by a single pipe of uniform
diameter D, but of the same length L=L
1
+ L
2
+ L
3
such that the head loss due to friction
for both the pipes, viz equivalent pipe & the compound pipe are the same.
For a compound pipe or pipes in series.
for an equivalent pipe
D
1
D
2
D
3
Q Q
Q
1
Q
2
Q
3
L = L
1
= L
2
= L
3
Pipes in Parallel
D
2
D
1
D
3
== Q D
L= L
1
+L
2
+L
3
L
1
L
3
Q
3 2 1
hf hf hf h
f
+ +
) 1 (
8 8 8
5
3
2
2
3
5
2
2
2
2
5
1
2
2
1
+ +
D g
Q fL
D g
Q fL
D g
Q fL
h
f

) 2 (
8
5
1
2
2

D g
fLQ
h
f

Equating (1) & (2) and simplifying


Or
5
3
3
5
2
2
5
1
1
5
D
L
D
L
D
L
D
L
+ +
5
1
5
3
3
5
2
2
5
1
1

'

+ +

D
L
D
L
D
L
L
D
Problems
1) Find the diameter of a Galvanized iron pipe required to carry a flow of 40lps of water, if the loss of head
is not to exceed 5m per 1km. Length of pipe, Assume f=0.02.
Solution:-
D=?, Q=40lps = 40x10
-3
m
3
/s
h
f
=5m, L=1km = 1000m. f=0.02
Darcys equation is
2) Two tanks are connected by a 500mm diameter 2500mm long pipe. Find the rate of
flow if the difference in water levels between the tanks is 20m. Take f=0.016. Neglect
minor losses.
Solution:-
Applying Bernoullis equation between (1) & (2) with (2) as datum & considering head
loss due to friction h
f
only,
Z
1
= 20m, Z
2
= 0 (Datum); V
1
=V
2
= 0 (tanks are very large)
p
1
=p
2
=0 (atmospheric pressure)
Therefore From (1)
20+0+0=0+0+0+h
f
Or, h
f
= 20m. But
mm m D 220 22 . 0
5
2
8
D g
fLQ
h
f

2
1
5 2
2500 016 . 0 8
5 . 0 81 . 9 20

'

x x
x x x
Q

lps m Q 8 . 434 sec / 4348 . 0
3

3) Water is supplied to a town of 0.5million inhabitants from a reservoir 25km away and
the loss of head due to friction in the pipe line is measured as 25m. Calculate the size of
the supply main, if each inhabitant uses 200 litres of water per day and 65% of the daily
supply is pumped in 8 hours. Take f=0.0195.
Solution:-
Number of inhabitants = 5million = 5,00,000
Length of pipe = 25km = 25,000m.
H
f
= 25m, D=?
Per capita daily demand = 200litres.
Total daily demand = 5,00,000x200= 100x10
6
litres.
Daily supply = 65/100 x 100x10
6
= 65,000m
3
.
Supply rate
4) An existing pipe line 800m long consists of four sizes namely, 30cm for 175m, 25cm
dia for the next 200m, 20cm dia for the next 250m and 15cm for the remaining length.
Neglecting minor losses, find the diameter of the uniform pipe of 800m. Length to
replace the compound pipe.
Solution:-
L=800m
L
1
=175m D
1
=0.3m
L
2
=200m D
2
=0.25m
L
3
=250m D
3
=0.20m
L
4
=175m D
4
=0.15m
For an equivalent pipe
D = Diameter of equivalent pipe = 0.189m less than or equal to 19cm.

'

5 2
2
8
D g
fLQ
h
f

5
1
2
2
25 81 . 9
) 1248 . 2 ( 000 , 25 195 . 0 8

'

x x
x x x
D

m D 487 . 1

'

+ + +
5
4
4
5
3
3
5
2
2
5
1
1
5
D
L
D
L
D
L
D
L
D
L
5) Two reservoirs are connected by four pipes laid in parallel, their respective diameters
being d, 1.5d, 2.5d and 3.4d respectively. They are all of same length L & have the same
friction factors f. Find the discharge through the larger pipes, if the smallest one carries
45lps.
Solution:-
D
1
=d, D
2
=1.5d, D
3
=2.5d, D
4
=3.4d
L
1
=L
2
=L
3
=L
4
= L.
f
1
=f
2
=f
3
=f
4
=f.
Q
1
=45x10
-3
m
3
/sec, Q
2
=? Q
3
=? Q
4
=?
For pipes in parallel hf
1
=hf
2
=hf
3
=hf
4 ,
i.e.
5
1
5 5 5 5
15 . 0
175
2 . 0
250
25 . 0
200
3 . 0
175
800

'

,
_

+ + +
D
5
4
2
4
5
3
2
3
5
2
2
2
5
1
2
1
D
Q
D
Q
D
Q
D
Q

( ) sec / 124 . 0 10 45
5 . 1
3
2
1
2
3
5
2
m x x
d
d
Q

'

,
_


( ) sec / 4446 . 0 10 45
5 . 2
3
2
1
2
3
5
2
m x x
d
d
Q

'

,
_


( ) sec / 9592 . 0 10 45
4 . 3
3
2
1
2
3
5
2
m x x
d
d
Q

'

,
_


6) Two pipe lines of same length but with different diameters 50cm and 75cm are made
to carry the same quantity of flow at the same Reynolds number. What is the ratio of
head loss due to friction in the two pipes?
Solution:-
D
1
=0.5m, D
2
=0.75m
L
1
=L
2
Q
1
=Q
2
(Re)
1
= (Re)
2
,
Reynolds number Re=
7) A 30cm diameter main is required for a town water supply. As pipes over 27.5cm
diameter are not readily available, it was decided to lay two parallel pipes of same
diameter. Find the diameter of the parallel pipes which will have the combined discharge
equal to the single pipe. Adopt same friction factor for all the pipes.
Solution:-

2 2 2
V D
2
2 2 2
1
1 1 1

V D V D

2 2 1 1
D V D V
2 1
75 . 0 5 . 0 V V
( )
2 1
( )
2 1

2 1
5 . 1 V V
gD
fLV
h
f
2
2

2
2
2
1
1
2
2
1
V
V
x
D
D
hf
hf

375 . 3
5 . 1
5 . 0
75 . 0
2
2
2

,
_

V
V
x
From Darcys equation
) 1 (
8
5 2
2

'

D g
fLQ
h
f

) 2 (
2
8
2
5 2

'

,
_

D g
Q
fL
h
f

Equating
8) Two reservoirs are connected by two parallel pipes. Their diameter are 300mm &
350mm and lengths are 3.15km and 3.5km respectively of the respective values of
coefficient of friction are 0.0216 and 0.0325. What will be the discharge from the larger
pipe, if the smaller one carries 285lps?
Solution:-
D
1
=300mm=0.3m, D
2
=-.350m
L
1
=3150m L
2
=3500m
F
1
=0.0216 f
2
=0.0325
Q
1
=0.285m
3
/sec Q
2
=?
For parallel pipes
5
1
5
4
275 . 0

'

D
m m D 275 . 0 205 . 0
or

'

'

5
2
2
2
2 2 2
5
1
2
2
1 1 1
8 8
D g
Q L f
D g
Q L f
h
f

2
1
5
1 2 2
5
2
2
1 1 1
2

'


D L f
D Q L f
Q
2
1
5
5 2
2
3 . 0 3500 0325 . 0
35 . 0 285 . 0 3150 0216 . 0

'


x x
x x x
Q
sec / 324 . 0
3
2
m Q
9) Consider two pipes of same lengths and having same roughness coefficient, but with
the diameter of one pipe being twice the other. Determine (I) the ratio of discharges
through these pipes, if the head loss due to friction for both the pipes is the same. (ii) the
ratio of the head loss due to friction, when both the pipes carry the same discharge.
Solution:-
f
1
=f
2
D
1
=2D
2
L
1
=L
2
(i)Given hf
1
=hf
2
Q
1
/Q
2
=?
From Darcys equation
(ii) Given Q
1
/Q
2,
hf
1
/hf
2
=?
10) Two sharp ended pipes are 50mm & 105mm diameters and 200m length are
connected in parallel between two reservoirs which have a water level difference of 15m.
If the coefficient of friction for each pipes of 0.0215. Calculate the rate of flow in each
pipe and also diameter of a single pipe 200m long which would give the same discharge,
if it were substituted for the Original two pipes.
Solution
D
1
=0.015m, D
2
=0.105m, L
1
=L
2
=200m
H=15m, f
1
=f
2
=0.0215,
a) Q
1
=?, Q
2
=?
(b) D=?, when Q=Q
1
+Q
2
a) For parallel pipes
656 . 5
2
2
5
2
2
2
5
2
1
2
1

,
_

,
_

D
D
D
D
Q
Q
03125 . 0
2
8
5
2
2
5
1
2
5
1
2
2
2 1 1
2
1

,
_

,
_


D
D
D
D
D g
Q L f
hf
hf

'

'

5
2
2
2
2 2 2
5
1
2
2
1 1 1
8 8
D g
Q L f
D g
Q L f
h
f

sec / 10 63 . 3
200 0215 . 0 8
05 . 0 81 . 9 15
3 3
2
1
5 2
1
m x
x x
x xh x
Q

'

sec / 023 . 0
200 0215 . 0 8
105 . 0 81 . 9 15
3 2
2
1
5 2
1
m
x x
x xh x
Q

'

( ) sec / 02684 . 0 0232 . 0 10 63 . 3


2 3
2 1
m x Q Q Q + +

b)
5 2
2
8
D g
fLQ
h
f

( )
5
1
2
2
15 81 . 9
02684 . 0 200 0215 . 0 8

'


x x
x x x
D

cm m D 12 . 11 1112 . 0
11) Two pipes with diameters 2D and D are first connected in parallel and when a
discharge Q passes the head loss is H
1
, when the same pipes are Connected in series for
the same discharge the loss of head is H
2
. Find the relationship between H
1
and H
2
.
Neglect minor losses. Both the pipes are of same length and have the same friction
factors.
Solution
H
1
= head loss due to friction = h
f
= hf
2
i.e.
Case(iii)
12) Two reservoirs are connected by a 3km long 250mm diameter. The difference in
water levels being 10m. Calculate the discharge in lpm, if f=0.03. Also find the
percentage increase in discharge if for the last 600m a second Pipe of the same diameter
is laid parallel to the first.
Solution
Applying Bernoullis equation between (1) & (2) with (2) as datum and
considering head loss due to friction h
f
5 2
2
5 2
2
) 2 (
8
8
D g
fLQ
D g
fLQ
h
f

+

'

,
_

+
5 5 2
2
2
2
1
1
1 8
D g
fLQ
H

) 4 (
8 0312 . 1
5 2
2
2

D g
xflQ x
H

2
5 2
5 2
2
2
1
8 0312 . 1
8 02256 . 0
flQ x
D g
x
D g
xflQ x
H
H


021876 . 0
0312 . 1
02256 . 0
2
1

H
H
71 . 45
1
2

H
H
Or
f
h
g
V p
Z
g
V p
Z + + + + +
2 2
2
1 2
2
2
1 1
1

m h h
f f
10 0 0 0 0 0 10 + + + + +
5 2
2
8
D g
fLQ
h
f

Case (ii)
Change in discharge =

( )
2
1
5 2
3000 03 . 0 8
10 25 . 0 81 . 9

'


x x
x x x
Q

sec / 03624 . 0
3
m Q
3 2 1
orhf hf hf h
f
+
( )

'

+
5
2
1
5
2
1
2
25 . 0
2 / 600
25 . 0
2400
81 . 9
03 . 0 8
10
Q Q
x
x

2
1
66 . 6472 10 Q
sec / 0393 . 0
3
1
m Q
( ) Q Q Q
1
( ) 03624 . 0 0393 . 0
sec / 10 066 . 3
3 3
m x Q


100
1
x
Q
Q
% increase in discharge =
% 46 . 8 100
03624 . 0
10 066 . 3
3

x
x
MINOR LOSSES IN PIPES
Minor losses in a pipe flow can be either due to change in magnitude or direction of flow.
They can be due to one or more of the following reasons.
i)Entry loss
ii)Exit loss
iii)Sudden expansion loss
iv)Sudden contraction loss
v)Losses due to pipe bends and fittings
vi)Losses due to obstruction in pipe.
Equation for head loss due to sudden enlargement or expansion of a pipe
Consider the sudden expansion of flow between the two section (1) (1)& (2) (2) as
shown.
P
1
& P
2
are the pressure acting at (1) (1) and (2) (2), while V
1
and V
2
are the velocities.
From experiments, it is proved that pressure P
1
acts on the area (a
2
a
1
) i.e. at the point
of sudden expansion.
From II Law of Newton Force = Mass x Acceleration.
Consider LHS of eq(1)
Consider RHS of eq(1)
Mass x acceleration = x vol x change in velocity /time
=volume/time x change in velocity
Substitution (ii) & (iii) in eq(i)
Both sides by (sp.weight)
Applying Bernoullis equation between (1) and (2) with the centre line of the pipe as
datum and considering head loss due to sudden expansion h
L
only.
( ) ) (
2 1
iii V V xQx
( ) ( )
2 1 2 1 2
V V pQ p p a
( ) ( )
2 1 2 2 1
V V V p p
or
zontal pipeishori C Z Z
2 1

In Eq(V) h
L
is expressed in meters similarly, power (P) lost due to sudden expansion is
Equations for other minor losses

Loss due to entrance and exit
Loss due to bends & fittings
Problems
g
V p
Z
g
V p
Z
2 2
2
2 2
2
2
1 1
1
+ + + +

( ) ( )
g
V V V V V
h
L
2
2
2
2
2
1 2 1 2
+

g
V V V V V
h
L
2
2 2
2
2
2
1
2
2 2 1
+

g
V V V V V
h
L
2
2 2
2
2
2
1 2 1
2
2
+

g
V V V V
h
L
2
2
2 1
2
1
2
2
+

( )
g
V V
h
L
2
2
2 1

g
V
h
L
2
5 . 0
2
2
Sudden contraction loss
g
V
h
exit L
2
2

g
KV
h
L
2
2

K=coefficient
1) A 25cm diameter, 2km long horizontal pipe is connected to a water tank. The pipe
discharges freely into atmosphere on the downstream side. The head over the centre line
of the pipe is 32.5m, f=0.0185. Considering the discharge through the pipe
Applying Bernoullis equation between (A) and (B) with (B) as datum & considering all
losses.
2) The discharge through a pipe is 225lps. Find the loss of head when the pipe is
suddenly enlarged from 150mm to 250mm diameter.
Solution :
D
1
=0.15m, D
2
= 0.25m Q=225lps = 225m
3
/sec
Head loss due to sudden expansion is
3) The rate of flow of water through a horizontal pipe is 350lps. The diameter of the pipe
is suddenly enlarge from 200mm to 500mm. The pressure intensity in the smaller pipe is
15N/cm
2
. Determine (i) loss of head due to sudden enlargement. (ii) pressure intensity in
the larger pipe (iii) power lost due to enlargement.
Solution
Q=350lps=0.35m
3
/s
D
1
=0.2m, D
2
=0.5m, P
1
=15N/cm
2
h
L
=?, p
2
=?, P=?
From continuity equation
( )
g
V V
h
L
2
1 2

g
X
D
Q
D
Q
2
1 4 4
2
2
2
1

,
_



2
2
2
2
1
2
2
1 1
2
16

,
_


D D g
Q

m h
L
385 . 3
2
2 2 2
2
25 . 0
1
15 . 0
1
81 . 9 2
225 . 0 16

,
_


x x
x
Applying Bernoullis equation between (1) (1) and (2) (2) with the central line of
the pipe as datum and considering head loss due to sudden expansion h
L
only.
4) At a sudden enlargement of an horizontal pipe from 100 to 150mm, diameter, the
hydraulic grade line raises by 8mm. Calculate the discharge through the pipe system.
Solution
Applying Bernoullis equation between (1) & (2) with the central line of the pipe as
datum and neglecting minor losses (h
L
) due to sudden expansion.
From continuity equation
L
h
g
V p
Z
g
V p
Z + + + + +
2 2
2
2 2
2
2
1 1
1

( ) ntal pipehorizo Z Z 0
2 1

463 . 4
62 . 19
78 . 1
81 . 9
0
62 . 19
14 . 11
81 . 9
150
0
2
2
2
+ + + + +
p
2 2
2
/ 67 . 16 / 68 . 166 cm N m kN p
L
Qh P
463 . 4 35 . 0 81 . 9 x x
kW P 32 . 15
( )
) 1 (
2
2
2 1

g
V V
h
L
) 2 ( 10 8 ,
3 1
1
2
2

1
]
1

,
_

,
_

+

m x
p
Z
p
Z Given

L
h
g
V p
Z
g
V p
Z + + + + +
2 2
2
2 2
2
2
1 1
1

Discharge
5) Two reservoirs are connected by a pipe line which is 125mm diameter for the first 10m
and 200mm in diameter for the remaining 25m. The entrance and exit are sharp and the
change of section is sudden. The water surface in the upper reservoir is 7.5m above that
in the lower reservoir. Determine the rate of flow, assuming f=0.001 for each of the
types.
Solution
From continuity equation
Applying Bernoullis equation between (1) & (2) in both the reservoirs with the water in
the lower reservoir as datum and considering all losses
2
2
1
2
4
15 . 0
4
1 . 0
xV
x
V
x

2 1
25 . 2 V V
s m
x
V / 25 . 0
1274 . 0
10 8
2
1
3
2

,
_

25 . 0
4
15 . 0
4
2
2
2
2
x
x
V
D
Q


2
2
1
2
4
2 . 0
4
125 . 0
V
x
V
x

2 1
56 . 2 V V
{ } 1 434 . 2 243 . 5 2768 . 3
62 . 19
5 . 7
2
2
+ + +
V
( ) ( ) ( )

'

+ + + +
g
V
g
V V
g
V x x
g
V
2 2
56 . 2
2
56 . 2 10 01 . 0
2
5 . 2 5 . 0
} 0 0 5 . 7
2
2
2
2
2
2
2
1
FLOW MEASUREMENTS
Flow Through Orifices
An orifice is an opening of any cross section, at the bottom or on the side walls of a
container or vessel, through which the fluid is discharged. If the geometric characteristics
of the orifice plus the properties of the fluid are known, then the orifice can be used to
measure the flow rates.
Classification of orifices
Flow through an orifice
As the fluid passes through the orifice under a head H, the stream lines converge and
therefore the jet contracts. The stream lines which converge are mostly those from near
the walls and they do so because stream lines cannot make right angled bend in motion.
This phenomenon occurs just down stream of the orifice, and such a section where the
area of cross section of the jet is minimum is know as VENA CONTRACTA.
The pressure at Vena Contracta is assumed to be atmospheric and the velocity is assumed
to be the same across the section since the stream lines will be parallel and equally
spaced. Downstream of Vena contracta the jet expands and bends down. Figure shows
the details of free flow through a vertical orifice.
Applying Bernoulli's equation between (B) & (C) with the horizontal through BC as
datum and neglecting losses (h
L
)
( ) s m V / 6 . 4 16 . 21 2
1
2

sec / 1445 . 0 6 . 4
4
2 . 0
2
2
m x
x
Q

'


Based on shape circular
triangular rectangular
Based on size
Small orifice
(when the head
over the orifice is
more than five
times its size I.e.
H>5d, Large
orifice
Based on shape of
the u/s edge
Sharp edge
Bell mouth
Based on
flow Free
Submerge
d
V V V
2 1
, 0
Velocity V in Eq(1) is known as TORRICELLIS VELOCITY.
Hydraulic Coefficients of an orifice
i)Coefficient of discharge (C
d
): It is defined as the ratio of actual discharge (Q
act
) to the
theoretical discharge (Q
th
)
Value of C
d
varies in the range of 0.61 to 0.65
ii) Coefficient of Velocity (C
v
): It is defined as the ratio of actual velocity (V
act
) to the
theoretical velocity (V
th
).
Value of C
v
varies in the range of 0.95 to 0.99
Coefficient of Contraction (C
c
): It is defined as the ratio of the area of cross section of
the jet at Vena of cross section of the jet at Vena Contracta (a
c
) to the area of the orifice
(a).
Value of C
c
will be generally more than 0.62.
Relationship between the Hydraulic Coefficients of an orifice
From continuity equation
Actual discharge Q
act
= a
c
x V
act
Theoretical discharge Q
th
= a x V
th
Equation for energy loss through an orifice
Applying Bernoullis equation between the liquid surface (A) and the centre of jet and
Vena Contracta (C) and considering losses (h
L
).

,
_


th
act
V
V
V
C

,
_


a
a
C
c
C
Torricellis equation
Equation for Coefficient of Velocity (C
V
) (Trajectory method)
Consider a point P on the centre line of the jet, such that its horizontal and vertical
coordinates are x and y respectively.
By definition, velocity
Since, the jet falls through a vertical distance y under the action of gravity during this
time (t)
Equating equations (1) & (2)
, H Z
A

, 0
A
V
) ( 0 city actualvelo p p
B A

L
h
g
Va
H + + + + +
2
0 0 0 0
2
)
2
(
2
g
Va
H h
L

gH C ButV
V a
2
) (
2
V L
HxC H h
) 1 (
2
V L
C H h
t
x
V
a

a
V
x
t
Or
Or
But
2
1
2

,
_

g
y
V
x
a
gH C V
V a
2
2
1
2
2

,
_

g
y
gH C
x
V
2
1
2
1
2
1
2
1
2
1
2
1
2 2 y
g
x
H g
x
C
V

Hy
x
C
V
2

1
1
]
1

yH
x
C
V
4
2
Or
Problems
1. The head of water over the centre of an orifice 30mm diameter is 1.5m. If the
coefficient of discharge for the orifice is 0.613, Calculate the actual discharge.
Solution:
d=30mm = 3x10
-3

H=1.5m
C
d
=0.613
2. Compensation water is to be discharge by two circular orifices under a constant head
of 1.0m, measured from the centre of the orifices. What diameter will be required to give
a discharge of 20x10
3
m
3
per day? Assume C
d
for each notch as 0.615.
Solution: d=? H=1m. Q
total
= 20x10
3
m
3
/day C
d
=0.615.
we know
3. A jet of water issuing from an orifice 25mm diameter under a constant head of 1.5m
falls 0.915m vertically before it strikes the ground at a distance of 2.288m measured
;
th
act
d
Q
Q
C
gH xa C
d
2
s m x Q
act
/ 10 35 . 2
3 3

lps Q
act
35 . 2
s m / 1157 . 0
3

gH a C Q
d act
2
1 81 . 9 2
4
615 . 0 1157 . 0
2
x x x
xd
x

mm m d 5 . 232 2325 . 0
horizontally from the Vena Contracta. The discharge was found to be 102lpm. Determine
the hydraulics coefficients of the orifice and the head due to resistance.
Solution: d=25mm=25x10
-3
H=1.5m, y=0.915m, x=2.288m
Q
act
=102lpm = 102/60 = 1.7lps = 1.7x10
-3
m
3
/sec, C
d
=?, C
c
=?, h
L
=?
4. The head of water over a 100mm diameter orifice is 5m. The water coming out of the
orifice is collected in a circular tank 2m diameter. The time taken to collect 45cm of
water is measured as 30secs. Also the coordinates of the jet at a point from Vena Contract
are 100cm horizontal and 5.2cm vertical. Calculate the hydraulic coefficients of the
orifice.
Solution:
D=100mm=0.1m, H=5m
Q
act
= Area of collecting tankxheight of water collected / time
X=100cm = 1m, y=5.2cm = 0.052m
C
d
=?, C
v
=?, C
c
=?
5. The coordinates of a point on the jet issuing from a vertical orifice are 0.4m & 0.003m.
Neglecting air resistance, determine the velocity of the jet and the height of water above
the orifice in the tank.
Solution.
s m x
x
/ 0471 . 0
30
45 . 0
4
2
3
2


98 . 0
5 052 . 0 4
1
4
2 2

,
_

,
_

x x yH
x
C
v
605 . 0
5 81 . 9 2 1 . 0
4 0471 . 0
2

'

,
_


x x x
x
Q
Q
C
th
act
d

618 . 0
98 . 0
605 . 0

V
d
C
C
C
C
X=0.4m, y=0.3m, V=? H=?
Assume
We know
6. A vertical orifice is fitted 0.2m above the bottom of a tank containing water to a depth
of 2m. If G=0.98. What is the vertical distance from the orifice of a point on the jet 0.6m
away from the Vena Contracta?
Solution
Head over the orifice H=(2-0.2)=1.8m
C
V
=0.98, y=?, x=0.6m
7. A closed tank contains water to a height of 2m above a sharp edged orifice 1.5cm
diameter, made in the bottom of the tank. If the discharge through the orifice is to be 4lps.
1
V
C
yH
x
C
V
4
2

,
_

2
2
2
2
2 2
1 03 . 0 4
4 . 0
4
4
x x yxG
x
H
x yHxG
H=1.33m
mm m
x x
y
xyx
or
yH
x
C
V
52 052 . 0
98 . 0 8 . 1 4
6 . 0
8 . 1 4
6 . 0
) 98 . 0 ( ,
4
2
2
2
2
2

,
_

Workout the pressure at which air should be pumped into the tank above water. Take
C
d
=0.6.
Solution
Q=4lps = 4x10
-3
m
3
/s
D=1.5x10
-2
m, C
d
=0.6
P
A
=?
Total head over the orifice
8. A closed tank contains 3m depth of water and an air space at 15kpa pressure. A 5cm
diameter orifice at the bottom of the tank discharge water to the tank B containing
pressurized air at 25kpa. If C
d
= 0.61 for the orifice. Calculate the discharge of water
from tank A.
Solution
d=5cm = 5x10
-2
m C
d
=0.61.
Total head over the orifice
H=1.9806m
9. A tank has two identical orifices in one of its vertical sides. The upper orifice is 4m
below the water surface and the lower one 6m below the water surface. If the value of C
v
for each orifice is 0.98, find the point of intersection of the two jets.
Solution.
3 3 3
/ 10 772 . 11 / 772 . 11 m kN x m N
air

,
_

A
p
h H
gH a C Q
d act
2
( )

,
_

3
2
2
3
10 772 . 11
2 81 . 9 2
4
10 5 . 1
6 . 0 10 4
x
P
x x x
x
x x x
A

) ( / 83 . 0
2
Gauge m kN P
A

( )

'

+

'

+
81 . 9
25 15
3

B A
p p
h H
Given C
v
is same for both the orifices
from figure
Substituting eq(1) in eq(2) and simplifying
Again
10. Two orifices have been provided in the side of the tank, one near the bottom and the
other near the top. Show that the jets from these two orifices will intersect a plane
through the base at the same distance from the tank if the head on the upper orifice is
equal to the height of the lower orifice above the base. Assume C
v
to be the same for
both the orifices.
2 2
2
2
1 1
2
1
4 4 H y
x
H y
x

) (
4 4
2 1
2 2
2
1 1
2
1
x x
H y
x
H y
x

) 1 ( 5 . 1 6 4
2 1 2 1
y ory y y
( )
) 2 ( 2
4 6
2 1
2 1
+
+
y y
y y
m y
y
y y
4
2 5 . 0
2 5 . 1
2
2
2 2

+
gives
H y
x
C
V
2 2
2
2
4

m x
x x
x
6 . 9
6 4 4
98 . 0
2
2
2

(points of intersection of the jets from the


Vena contracts)
Solution.
To show that x
1
=x
2
when H
1
=y
2
from figure y
1
=[y
2
+(H
2
-H
1
)---(1)
Problems on Orifices
A 4cm dia orifice in the vertical side of a tank discharges water. The water surface in the
tank is at a constant level of 2m above the centre of the orifice. If the head loss in the
orifice is 0.2m and coefficient of contraction can be assumed to be 0.63. Calculate (I) the
values of coefficient of velocity & coefficient of discharge, (ii) Discharge through the
orifice and (iii) Location of the point of impact of the jet on the horizontal plane located
0.5m below the centre of the orifice.
Solution
Head loss
2 2
2
2
1 1
2
1
1
4 4
,
2
H y
x
H y
x
C C Given
V
V

Or
Or
2 2
2
1 2 1 1 2
H y H H H H y +
0 ) (
1 2 2 2 1
2
1
+ H H y H H H
0 0
0 ) ( ;
2 2 2 2 2
2
2 2 1

+ y H y y H y y H
substituting

,
_


g
Va
H h
L
2
2

,
_


81 . 9 2
2 2 . 0
2
x
Va Or
Coefficient of Velocity
Coefficient of discharge
(ii) Discharge through the orifice
(iii) Coefficient of velocity
An orifice has to be placed in the side of a tank so that the jet will be at a maximum
horizontal distance at the level of its base. If the depth of the liquid int the tank is D, what
is the position of the orifice? Show that the jets from the two orifices in the side of the
tank will intersect at the level of the base if the head on the on the upper orifice is equal
to the height of the orifice above the base.
Solution:
943 . 0
246 . 6
943 . 5

V
V
C
a
v
C v d
xC C C 63 . 0 949 . 0 x
598 . 0
d
C
gH a C Q
d act
2
2 81 . 9 2 04 . 0
4
598 . 0
2
x x x x x

yH
x
C
v
4
2

2
4 x yHC
V

x
Jet
Orifice
D
h
y
By definition, Velocity V=x/t
But
and

For x to be maximum
Vt
gH V 2
2
2
1
gt y
2
2 2
1
) (

,
_


gH
x
g H D
( ) H D H x 4
2
) ( 4 H D H x
Or
0
dH
dx
0 ) 2 ( 4 H D
2 / D H
We know, x=Vt,
1
2gH V
2
2
2
1
gt H y +
2
1
2 2
1
1
1
]
1

gH
x
g
FLOW THROUGH PIPES
Definition of flow through pipes
A pipe is a closed conduit carrying a fluid under pressure. Fluid motion in a pipe
is subjected to a certain resistance. Such a resistance is assumed to be due to
Friction. In reality this is mainly due to the viscous property of the fluid.
Reynolds Number (Re)
It is defined as the ratio of Inertia force of a flowing fluid and the Viscous force.
Re=(Inertia force/Viscous force) =( V D/ )
Classification of pipe flow:
Based on the values of Reynolds number (Re), flow is classified as
Follows:
Laminar flow or Viscous Flow
In such a flow the viscous forces are more predominent compared to inertia
Forces. Stream lines are practically parallel to each other or flow takes place
In the form of telescopic tubes.
This type of flow occurs when Reynolds number Re< 2000.
In laminar flow velocity increases gradually from zero at the boundary to
Maximum at the center.
Laminar flow is regular and smooth and velocity at any point practically remains
constant in magnitude & direction. Therefore, the flow is also known as stream
Line flow.
There will be no exchange of fluid particles from one layer to another.
Thus there will be no momentum transmission from one layer to another.
Ex: Flow of thick oil in narrow tubes, flow of Ground Water, Flow of
Blood in blood vessels.
Transition flow:
In such a type of flow the stream lines get disturbed a little.
This type of flow occurs when 2000< Re < 4000.
Turbulent Flow: This is the most common type of flow that occurs in nature( flow in rivers, pipes).
This flow will be random,erratic,unpredictable. Thus motion of fluid particles result in eddy currents
& they mix up. Streamlines are totally disturbed or cross each other.
The velocity changes in direction and magnitude from point to point.
There will be transfer of momentum between the particles as they are continuously colliding with
each other.
There will be considerable loss of energy in this type of flow.
This type of flow cannot be truly mathematically analysed and any analysis is possible by stastical
evaluation.
For this type of flow in a pipe Re> 4000.
(REYNOLDS EXPERIMENT:Refer Fig.(1)
Hydraulic Grade Line & Energy Grade Line
A Line joining the peizometric heads at various points in a flow is known as Hydraulic
Grade Line (HGL)
Energy Grade Line (EGL)
It is a line joining the elevation of total energy of a flow measured above a datum, i.e.
.
2
2
g
V p
Z + +

EGL Line lies above HGL by an amount V


2
/2g.(Refer Fig.(2))
Losses in Pipe Flow
Losses in pipe flow can be two types viz:-
a)Major Loss
b)Minor Loss
a)Major Loss: As the name itself indicates, this is the largest of the losses in a pipe. This loss
occurs due to friction only. Hence, it is known as head loss due to friction (h
f
)
b)Minor Loss: Minor losses in a pipe occurs due to change in magnitude or direction of flow.
Minor losses are classified as (i) Entry Loss, (ii) Exit loss, (iii) Sudden expansion loss (iv)
Sudden contraction loss (v) Losses due to bends & pipe fittings.
Head Loss due to Friction (DARCY-WEISBACH Equation)
Consider the flow through a straight horizontal pipe of diameter D, Length L, between two sections (1) & (2)
as shown in fig.(3). Let P
1
& P
2
be the pressures at these sections. o is the shear stress acting along the
pipe boundary.
From II Law of Newton
Force = Mass x accn. But acceleration = 0, as there is no change in velocity,
however the reason that pipe diameter is uniform or same throughout.
( )
( ) ) 1 (
4
4
4 4
. .
0
0
2 1
0
2
2 1
0
2
2
2
1


+

D
L
P P or
DL
D
P P
DL x
D
P
D
P e i
forces



Applying Bernoullis equation between (1) & (2) with the centre line of the pipe
as datum & considering head loss due to friction h
f
,.
f
h
g
V p
Z
g
V p
Z + + + + +
2 2
2
2 2
2
2
1 1
1

2 1
Z Z
Pipe is horizontal

2 1
V V
Pipe diameter is same throughout
) 2 (
2 1

f
h
P P

Substituting eq (2) in eq.(1)


) 3 (
4
4
0
0

l
D h
or
D
L
x h
f
f

From Experiments, Darcy Found that


) 4 (
8
2
0
V
f

f=Darcys friction factor (property of the pipe materials
Mass density of the liquid.
V = velocity
Equations (3) & (4)
L
D h
V
f
f
4 8
2


or ,
D
V Lf
h
f

8
4
2

But,
) 5 (
2
2

,
_

gD
fLV
h
g
f

from Continuity equation


2
4
D
Q
V

) 6 (
8
5 2
2

,
_


D gh
fLQ
h
f
& (5) & (6) are known as DARCY WEISBACH Equation
Pipes in Series or Compound Pipe
D
1
, D
2
, D
3
, D
4
are diameters.(fig.4)
L
1
, L
2
,L
3
, L
4
are lengths of a number of Pipes connected in series
(hf)
1
, (hf)
2
, (hf)
3 &
(hf)
4
are the head loss due to friction for each pipe.
The total head loss due to friction h
f
for the entire pipe system is given by
4 3 2 1
hf hf hf hf h
f
+ + +
5
4
2
2
4
5
3
2
2
3
5
2
2
2
2
5
1
2
2
1
8 8 8 8
D g
Q fL
D g
Q fL
D g
Q fL
D g
Q fL
h
f

+ + +
Pipes in parallel
D
1
, D
2
and D
3
are the pipe diameters.(Fig.5)
Length of each pipe is same, that is, L
1
=L
2
=L
3
For pipes in parallel hf1=hf2=hf3
i.e
) 1 (
8 8 8
5
3
2
5
2
2
5
1
2
5
3
2
2
3
5
2
2
2
2
5
1
2
2
1
3 2 1
3 2 1


D
Q
D
Q
D
Q
or
D g
Q fL
D g
Q fL
D g
Q fL

From continuity equation Q= Q
1
+Q
2
+Q
3
--------(2)
Equivalent pipe
In practice adopting pipes in series may not be feasible due to the fact that
they may be of unistandard size (ie. May not be comemercially available)
and they experience other minor losses. Hence, the entire system will be
replaced by a single pipe of uniform diameter D, but of the same length
L=L
1
+ L
2
+ L
3
such that the head loss due to friction for both the pipes, viz
equivalent pipe & the compound pipe are the same (Fig.6).
For a compound pipe or pipes in series
3 2 1
hf hf hf h
f
+ +
) 1 (
8 8 8
5
3
2
2
3
5
2
2
2
2
5
1
2
2
1
+ +
D g
Q fL
D g
Q fL
D g
Q fL
h
f

for an equivalent pipe ) 2 (
8
5
1
2
2

D g
fLQ
h
f

Equating (1) & (2) and simplifying


5
3
3
5
2
2
5
1
1
5
D
L
D
L
D
L
D
L
+ +
or
5
1
5
3
3
5
2
2
5
1
1

'

+ +

D
L
D
L
D
L
L
D
PROBLEMS
1) Find the diameter of a Galvanized iron pipe required to carry a flow of 40lps of water, if the loss
of head is not to exceed 5m per 1km. Length of pipe, Assume f=0.02.
Solution:-
D=?, Q=40lps = 40x10
-3
m
3
/s
h
f
=5m, L=1km = 1000m. f=0.02
Darcys equation is
5
2
8
D g
fLQ
h
f

'


f
h g
fLQ
D
2
2
8

5
1
2
2 3
5 81 . 9
) 10 40 ( 1000 02 . 0 8

'

x x
x x x x
D

mm m D 220 22 . 0
2) Two tanks are connected by a 500mm diameter 2500mm long pipe. Find the rate of flow if the
difference in water levels between the tanks is 20m. Take f=0.016. Neglect minor losses.
Solution:-
Applying Bernoullis equation between (1) & (2) with (2) as datum & considering head loss due to
friction h
f
only, (Fig.7).
) 1 (
2 2
2
2 2
2
2
1 1
1
+ + + + +
f
h
g
V p
Z
g
V p
Z

Z
1
= 20m, Z
2
= 0 (Datum); V
1
=V
2
= 0 (tanks are very large)
p
1
=p
2
=0 (atmospheric pressure)
Therefore From (1)
20+0+0=0+0+0+h
f
Or, h
f
= 20m.
But
5
2
8
D g
fLQ
h
f

2
1
5 2
2500 016 . 0 8
5 . 0 81 . 9 20

'

x x
x x x
Q

lps m Q 8 . 434 sec / 4348 . 0
3

3) Water is supplied to a town of 0.5million inhabitants from a reservoir 25km away and the loss
of head due to friction in the pipe line is measured as 25m. Calculate the size of the supply main,
if each inhabitant uses 200 litres of water per day and 65% of the daily supply is pumped in 8
hours. Take f=0.0195.
Solution:-
Number of inhabitants = 5million = 5,00,000
Length of pipe = 25km = 25,000m.
H
f
= 25m, D=?
Per capita daily demand = 200litres.
Total daily demand = 5,00,000x200= 100x10
6
litres.
Daily supply = 65/100 x 100x10
6
= 65,000m
3
.
Supply rate sec / 1248 . 2
60 60 5 . 8
000 , 65
3
m
x x
Q

'

'

5 2
2
8
D g
fLQ
h
f

5
1
2
2
25 81 . 9
) 1248 . 2 ( 000 , 25 195 . 0 8

'

x x
x x x
D

m D 487 . 1
4) An existing pipe line 800m long consists of four sizes namely, 30cm for 175m, 25cm dia for the
next 200m, 20cm dia for the next 250m and 15cm for the remaining length. Neglecting minor
losses, find the diameter of the uniform pipe of 800m. Length to replace the compound pipe.
Solution:-
L=800m
L
1
=175m D
1
=0.3m
L
2
=200m D
2
=0.25m
L
3
=250m D
3
=0.20m
L
4
=175m D
4
=0.15m
For an equivalent pipe

'

+ + +
5
4
4
5
3
3
5
2
2
5
1
1
5
D
L
D
L
D
L
D
L
D
L
5
1
5 5 5 5
15 . 0
175
2 . 0
250
25 . 0
200
3 . 0
175
800

'

,
_

+ + +
D
D = Diameter of equivalent pipe = 0.189m less than or equal to 19cm.
5) Two reservoirs are connected by four pipes laid in parallel, their respective diameters being d,
1.5d, 2.5d and 3.4d respectively. They are all of same length L & have the same friction factors f.
Find the discharge through the larger pipes, if the smallest one carries 45lps.
Solution:-
D
1
=d, D
2
=1.5d, D
3
=2.5d, D
4
=3.4d
L
1
=L
2
=L
3
=L
4
= L.
f
1
=f
2
=f
3
=f
4
=f.
Q
1
=45x10
-3
m
3
/sec, Q
2
=? Q
3
=? Q
4
=?
For pipes in parallel hf
1
=hf
2
=hf
3
=hf
4
i.e.
5
4
2
4
5
3
2
3
5
2
2
2
5
1
2
1
D
Q
D
Q
D
Q
D
Q

( ) sec / 124 . 0 10 45
5 . 1
3
2
1
2
3
5
2
m x x
d
d
Q

'

,
_


( ) sec / 4446 . 0 10 45
5 . 2
3
2
1
2
3
5
2
m x x
d
d
Q

'

,
_


( ) sec / 9592 . 0 10 45
4 . 3
3
2
1
2
3
5
3
m x x
d
d
Q

'

,
_


6) Two pipe lines of same length but with different diameters 50cm and 75cm are made to carry
the same quantity of flow at the same Reynolds number. What is the ratio of head loss due to
friction in the two pipes?
Solution:-
D
1
=0.5m, D
2
=0.75m
L
1
=L
2
Q
1
=Q
2
(Re)
1
= (Re)
2
,
?
2
1

hf
hf
Reynolds number Re=

V D
2
2 2 2
1
1 1 1

V D V D

2 2 1 1
D V D V ( )
2 1
( )
2 1

2 1
75 . 0 5 . 0 V V
2 1
5 . 1 V V
From Darcys equation
gD
fLV
h
f
2
2

2
2
2
1
1
2
2
1
V
V
x
D
D
hf
hf

375 . 3
5 . 1
5 . 0
75 . 0
2
2
2

,
_

V
V
x
7) A 30cm diameter main is required for a town water supply. As pipes over 27.5cm diameter are
not readily available, it was decided to lay two parallel pipes of same diameter. Find the diameter
of the parallel pipes which will have the combined discharge equal to the single pipe. Adopt
same friction factor for all the pipes.
Fig.(8)
Solution:-
) 1 (
8
5 2
2

'

D g
fLQ
h
f


) 2 (
2
8
2
5 2

'

,
_

D g
Q
fL
h
f

Equating

'

,
_

'

5 2
2
5 2
2
2
8
8
D g
Q
fL
D g
fLQ

5 5
1
4
1 1
D D

or
5
1
5
4
275 . 0

'

D
m m D 275 . 0 205 . 0
8) Two reservoirs are connected by two parallel pipes. Their diameter are 300mm & 350mm and
lengths are 3.15km and 3.5km respectively of the respective values of coefficient of friction are
0.0216 and 0.0325. What will be the discharge from the larger pipe, if the smaller one carries
285lps?
Solution:-
D
1
=300mm=0.3m, D
2
=-.350m
L
1
=3150m L
2
=3500m
F
1
=0.0216 f
2
=0.0325
Q
1
=0.285m
3
/sec Q
2
=?
For parallel pipes

'

'

5
2
2
2
2 2 2
5
1
2
2
1 1 1
8 8
D g
Q L f
D g
Q L f
h
f

2
1
5
1 2 2
5
2
2
1 1 1
2

'


D L f
D Q L f
Q
2
1
5
5 2
2
3 . 0 3500 0325 . 0
35 . 0 285 . 0 3150 0216 . 0

'


x x
x x x
Q
sec / 324 . 0
3
2
m Q
9) Consider two pipes of same lengths and having same roughness coefficient, but with the
diameter of one pipe being twice the other. Determine (I) the ratio of discharges through these
pipes, if the head loss due to friction for both the pipes is the same. (ii) the ratio of the head loss
due to friction, when both the pipes carry the same discharge.
Solution:-
f
1
=f
2
D
1
=2D
2
L
1
=L
2
(i)Given hf
1
=hf
2
Q
1
/Q
2
=?
From Darcys equation

'

5 2
2
8
D g
fLQ
h
f

5
2
2
2
2 2 2
5
1
2
2
1 1 1
8 8
D g
Q L f
D g
Q L f



656 . 5
2
2
5
2
2
2
5
2
1
2
1

,
_

,
_

D
D
D
D
Q
Q
(ii) Given Q
1
/Q
2,
hf
1
/hf
2
=?
03125 . 0
2
8
5
2
2
5
1
2
5
1
2
2
2 1 1
2
1

,
_

,
_


D
D
D
D
D g
Q L f
hf
hf

10) Two sharp ended pipes are 50mm & 105mm diameters and 200m length
are connected in parallel between two reservoirs which have a water level
difference of 15m. If the coefficient of friction for each pipes of 0.0215.
Calculate the rate of flow in each pipe and also diameter of a single pipe
200m long which would give the same discharge, if it were substituted for the
Original two pipes.
Solution
D
1
=0.015m, D
2
=0.105m, L
1
=L
2
=200m
H=15m, f
1
=f
2
=0.0215, a) Q
1
=?, Q
2
=? (b) D=?, when Q=Q
1
+Q
2
a) For parallel pipes

'

'

5
2
2
2
2 2 2
5
1
2
2
1 1 1
8 8
D g
Q L f
D g
Q L f
h
f

sec / 10 63 . 3
200 0215 . 0 8
05 . 0 81 . 9 15
3 3
2
1
5 2
1
m x
x x
x xh x
Q

'

sec / 023 . 0
200 0215 . 0 8
105 . 0 81 . 9 15
3 2
2
1
5 2
2
m
x x
x xh x
Q

'

b)
( ) sec / 02684 . 0 0232 . 0 10 63 . 3
2 3
2 1
m x Q Q Q + +

5 2
2
8
D g
fLQ
h
f


( )
5
1
2
2
15 81 . 9
02684 . 0 200 0215 . 0 8

'


x x
x x x
D


cm m D 12 . 11 1112 . 0
11) Two pipes with diameters 2D and D are first connected in parallel and
when a discharge Q passes the head loss is H
1
, when the same pipes are
Connected in series for the same discharge the loss of head is H
2
. Find the
relationship between H
1
and H
2
. Neglect minor losses. Both the pipes are of
same length and have the same friction factors.
Solution (Fig.9)
H
1
= head loss due to friction = h
f
= hf
2
i.e.

'

5 2
2
1
) 2 (
8
D g
fLQ
h
f

) 1 (
) (
8
5 2
2
2
2

'

D g
fLQ
h
f

) 2 (
2 1
+ Q Q Q
Q Q Q +
2 2
66 . 5 Q Q
66 . 6
1
2

) 3 (
02256 . 0 8 66 . 6
1
8
5 2
2
5 2
2
1

,
_


D g
Q fLx
D g
Q fL
H

Case(iii)
2 1 2
hf hf H +
5 2
2
5 2
2
) 2 (
8
8
D g
fLQ
D g
fLQ
h
f

+

'

,
_

+
5 5 2
2
2
2
1
1
1 8
D g
fLQ
H

) 4 (
8 0312 . 1
5 2
2
2

D g
xflQ x
H

2
5 2
5 2
2
2
1
8 0312 . 1
8 02256 . 0
flQ x
D g
x
D g
xflQ x
H
H


021876 . 0
0312 . 1
02256 . 0
2
1

H
H
or
71 . 45
1
2

H
H
12) Two reservoirs are connected by a 3km long 250mm diameter. The
difference in water levels being 10m. Calculate the discharge in lpm, if f=0.03.
Also find the percentage increase in discharge if for the last 600m a second
Pipe of the same diameter is laid parallel to the first.
Solution
Applying Bernoullis equation between (1) & (2) with (2) as datum and
considering head loss due to friction h
f (Fig.11)
f
h
g
V p
Z
g
V p
Z + + + + +
2 2
2
1 2
2
2
1 1
1

m h h
f f
10 0 0 0 0 0 10 + + + + +
5 2
2
8
D g
fLQ
h
f


( )
2
1
5 2
3000 03 . 0 8
10 25 . 0 81 . 9

'


x x
x x x
Q

sec / 03624 . 0
3
m Q
Case (ii)
3 2 1
orhf hf hf h
f
+

3 2 1
orhf hf hf h
f
+
(Fig.12)
( )

'

+
5
2
1
5
2
1
2
25 . 0
2 / 600
25 . 0
2400
81 . 9
03 . 0 8
10
Q Q
x
x

2
1
66 . 6472 10 Q sec / 0393 . 0
3
1
m Q
Change in discharge = ( ) Q Q Q
1
( ) 03624 . 0 0393 . 0 sec / 10 066 . 3
3 3
m x Q


% increase in discharge =
100
1
x
Q
Q
% 46 . 8 100
03624 . 0
10 066 . 3
3

x
x
MINOR LOSSES IN PIPES
Minor losses in a pipe flow can be either due to change in magnitude or direction
of flow. They can be due to one or more of the following reasons.
i)Entry loss
ii)Exit loss
iii)Sudden expansion loss
iv)Sudden contraction loss
v)Losses due to pipe bends and fittings
vi)Losses due to obstruction in pipe.
Equation for head loss due to sudden enlargement or expansion of a pipe
Consider the sudden expansion of flow between the two section (1) (1)& (2) (2)
as shown in Fig.13
P
1
& P
2
are the pressure acting at (1) (1) and (2) (2), while V
1
and V
2
are the
velocities.
From experiments, it is proved that pressure P
1
acts on the area (a
2
a
1
) i.e. at
the point of sudden expansion.
From II Law of Newton Force = Mass x Acceleration. ---------------(1)
Consider LHS of eq(1)
( ) ) (
1 2 1 2 2 1 1
i a a p a p a p forces + +

( ) ) ( ,
2 1 2
ii p p a forces or

Consider RHS of eq(1)


Mass x acceleration =

x vol x change in velocity /time

=volume/time x change in velocity


Substitution (ii) & (iii)
( ) ( )
2 1 2 1 2
V V Q p p a
or, ( ) ( )
2 1 2 2 1
V V V p p
Both sides by (sp.weight)
( )
) (
2 1 2 2 1
iv
g
V V V p p

,
_

Applying Bernoullis equation between (1) and (2) with the centre line of the pipe
as datum and considering head loss due to sudden expansion h
L
only
g
V p
Z
g
V p
Z
2 2
2
2 2
2
2
1 1
1
+ + + +

zontal pipeishori C Z Z
2 1

L
h
g
V V p p

,
_

2
2
2
2
1 2 1

( ) ( )
g
V V V V V
h
L
2
2
2
2
2
1 2 1 2
+

g
V V V V V
h
L
2
2 2
2
2
2
1
2
2 2 1
+

g
V V V V V
h
L
2
2 2
2
2
2
1 2 1
2
2
+

g
V V V V
h
L
2
2
2 1
2
1
2
2
+
OR
( )
g
V V
h
L
2
2
2 1

In Eq(V) h
L
is expressed in meters similarly, power (P) lost due to sudden
expansion is
) (vi Qh P
f

Equations for other minor losses (Fig.14 a,b,c)
Sudden contraction loss
g
V
h
L
2
5 . 0
2
2

Loss due to entrance and exit


g
V
h
entry L
2
5 . 0
2


g
V
h
exit L
2
2

Loss due to bends & fittings


g
KV
h
L
2
2

K=coefficient
Problems
1) A 25cm diameter, 2km long horizontal pipe is connected to a water tank. The
pipe discharges freely into atmosphere on the downstream side. The head over
the centre line of the pipe is 32.5m, f=0.0185. Considering the discharge through
the pipe
Applying Bernoullis equation between (A) and (B) with (B) as datum &
considering all losses.(Fig.15)
exitloss ss frictionlo entryloss
g
v p
Z
g
V P
Z
B B
B
A A
A
+ + + + + + +
2 2
2 2

g
V
gD
fLV
g
V
g
V
2 2 2
5 . 0
2
0 0 0 0 5 . 32
2 2 2 2
+ + + + + + +

'

+ + + 1
25 . 0
2000 0185 . 0
5 . 0 1
2
5 . 32
2
X
g
V
2
67 . 7 5 . 32 V
s m V / 06 . 2
4
2
D
Q

sec / 101 . 0 06 . 2
4
25 . 0
3
2
m x x

lps Q 101
2) The discharge through a pipe is 225lps. Find the loss of head when the pipe is
suddenly enlarged from 150mm to 250mm diameter.
Solution: D
1
=0.15m, D
2
= 0.25m Q=225lps = 225m
3
/sec
Head loss due to sudden expansion is
g
X
D
Q
D
Q
2
1 4 4
2
2
2
1

,
_




2
2
2
2
1
2
2
1 1
2
16

,
_


D D g
Q


2
2 2 2
2
25 . 0
1
15 . 0
1
81 . 9 2
225 . 0 16

,
_


x x
x

m h
L
385 . 3
3) The rate of flow of water through a horizontal pipe is 350lps. The diameter of
the pipe is suddenly enlarge from 200mm to 500mm. The pressure intensity in
the smaller pipe is 15N/cm
2
. Determine (i) loss of head due to sudden
enlargement. (ii) pressure intensity in the larger pipe (iii) power lost due to
enlargement.
Solution : (Fig.16)
Q=350lps=0.35m
3
/s
D
1
=0.2m, D
2
=0.5m, P
1
=15N/cm
2
h
L
=?, p
2
=?, P=?
From continuity equation
s m
x
x
D
Q
V / 14 . 11
2 . 0
35 . 0 4 4
2 2
1
1


s m
x
x
D
Q
V / 78 . 1
5 . 0
35 . 0 4 4
2 2
2
2


( ) ( )
mofwater
x g
V V
h
L
463 . 4
81 . 9 2
78 . 1 14 . 11
2
2
2 1

Applying Bernoullis equation between (1) (1) and (2) (2) with the central line of
the pipe as datum and considering head loss due to sudden expansion h
L
only.
Power lost
L
h
g
V p
Z
g
V p
Z + + + + +
2 2
2
2 2
2
2
1 1
1

( ) ntal pipehorizo Z Z 0
2 1

463 . 4
62 . 19
78 . 1
81 . 9
0
62 . 19
14 . 11
81 . 9
150
0
2
2
2
+ + + + +
p
2 2
2
/ 67 . 16 / 68 . 166 cm N m kN p
Power lost
L
Qh P 463 . 4 35 . 0 81 . 9 x x
kW P 32 . 15
4) At a sudden enlargement of an horizontal pipe from 100 to 150mm, diameter,
the hydraulic grade line raises by 8mm. Calculate the discharge through the pipe
system.
Solution
( )
) 1 (
2
2
2 1

g
V V
h
L
(Fig.17)
) 2 ( 10 8 ,
3 1
1
2
2

1
]
1

,
_

,
_

+

m x
p
Z
p
Z Given

Applying Bernoullis equation between (1) & (2) with the central line of the pipe as
datum and neglecting minor losses (h
L
) due to sudden expansion.
L
h
g
V p
Z
g
V p
Z + + + + +
2 2
2
2 2
2
2
1 1
1

0
2
2
1
2
2 1
1
2
2

'

,
_

,
_

+
L
h
g
V V p
Z
p
Z

From continuity equation
2
2
1
2
4
15 . 0
4
1 . 0
xV
x
V
x

2 1
25 . 2 V V
( ) ( )
0
81 . 9 2
25 . 2
81 . 9 2
25 . 2
10 8
2
2 2
2
2
2
2 3

'

+

x
V V
x
V V
x
0 1274 . 0 10 8
2
2
3

V x
s m
x
V / 25 . 0
1274 . 0
10 8
2
1
3
2

,
_

Discharge 25 . 0
4
15 . 0
4
2
2
2
2
x
x
V
D
Q


s m x / 10 428 . 4
3 3
or
lps Q 425 . 4
5) Two reservoirs are connected by a pipe line which is 125mm diameter for the
first 10m and 200mm in diameter for the remaining 25m. The entrance and exit
are sharp and the change of section is sudden. The water surface in the upper
reservoir is 7.5m above that in the lower reservoir. Determine the rate of flow,
assuming f=0.001 for each of the types.
Solution
From continuity equation
2
2
1
2
4
2 . 0
4
125 . 0
V
x
V
x


2 1
56 . 2 V V
Applying Bernoullis equation between (1) & (2) in both the reservoirs with the
water in the lower reservoir as datum and considering all losses
ansionloss sudden ss frictionlo entryloss
g
V p
Z
g
V p
Z
B B
B
A A
A
exp
2 2
2 2
+ + + + + + +

( )

'


+ + + + + + +
g
V V
g
V fL
g
V
2 2 2
5 . 0
0 0 0 } 0 0 5 . 7
2
2 1
2
1 1
2
1
( ) ( ) ( )

'

+ + + +
g
V
g
V V
g
V x x
g
V
2 2
56 . 2
2
56 . 2 10 01 . 0
2
5 . 2 5 . 0
} 0 0 5 . 7
2
2
2
2
2
2
2
1
{ } 1 434 . 2 243 . 5 2768 . 3
62 . 19
5 . 7
2
2
+ + +
V
( ) s m V / 6 . 4 16 . 21 2
1
2

sec / 1445 . 0 6 . 4
4
2 . 0
2
2
m x
x
Q

'


Additional Problems
Flow through Pipes
1) Water flows upwards through a vertical pipeline. A mercury manometer
connected between two points 10m apart shows a reading of 40cm of mercury
when discharge is 450lpm. If the friction factor is 0.02. Determine the size of the
pipe.
2) A town having a population of 1.2lakhs is to be supplied with water from a
reservoir 4km away, and it is stipulated that half the daily supply at the rate of
140lpcd should be delivered in 8 hours. Determine the size of the concrete pipes
to be laid, if the available head is 12m K for concrete pipes = 0.3mm.
3) Two reservoirs are connected by three pipes of same length laid in parallel,
and the diameters are D, 2D & 3D respectively. If the coefficients of friction of all
the three pipes is same, and the discharge in the smallest pipe is 30lps,
determine the flow rates in the other two pipes.
4) Two reservoirs are connected by a long pipes 300mm, diameter carrying
150lps. If another pipe of the same material is to be laid in parallel to carry twice
this discharge, what should be its diameter? Neglect minor losses.
5) Three pipes are connected in parallel between two points and the total
discharge is 3 cumecs. If the pipes are of length 1200m, 1400m & 1600m
diameter 1m, 0.8m & 1.2m respectively, and friction factor is the same for all the
pipes, determine the discharge in each pipe and the pressure difference required
to maintain the flow, assuming f=0.02.
6) A 450mm concrete pipe 1800m long connects two reservoirs whose difference
in water level is 15m. What is the discharge? If another concrete pipe line
300mm diameter is introduced in parallel what would be the percentage increase
in discharge and the discharge in each pipe. If the parallel pipe is introduced.
a)In the first half of the length. b)In the second half of the length
c)In the middle one third of the length. Assume f=0.03 for all pipes and same
difference in the reservoir levels.
7) A 450mm, concrete pipeline 200m long connects two reservoirs whose
difference in water levels is 15m. What is the discharge?
a)What is the percentage increase in discharge if another pipe line of the same
diameter is introduced is parallel for the second half of the length?
b)If a 30% increase in discharge is desired, what diameter pipe should be
introduced in parallel for the second half of the length? Assume f=0.03 for all the
pipes and the difference in reservoir levels same in the both the cases. Neglect
minor losses.
8) Two pipes of 5cm diameter and 10cm diameter are connected in series. They
have the same length and friction factor. If the head loss in the 10cm pipes is 1m,
what is the head loss in the 5cm pipe? If the discharge through the 10cm pipe is
10lps, what is the discharge through the 5cm pipe?
P1 V1
P2
V2
flow
Area = a1
Area = a2
9) A pipe has D=40cm, L=10m, f=0.02. What is the length of an equivalent pipe
which has D=20cm and f=0.02.
10) An 8cm diameter pipe carrying water has an abrupt expansion 12cm
diameter at a section. If a differential mercury manometer connected to upstream
and downstream sections of the expansion indicate a gauge reading of 2cm.
Estimate the discharge in the pipe.
11) When a sudden contraction form 50cm diameter to 25cm is introduced in a
horizontal pipe line the pressure changes from 105kps to 69kps. Assuming a
coefficient of contraction of 0.65, calculate the flow rate. Following this
contraction if there is a sudden enlargement to 50cm and if the pressure in the
25cm diameter section is 69kps, what is the pressure in the 50cm section?
12) Three pipes A, B & C with details as given in the following are connected in
series.
Calculate a) the size of a pipe of length 125m and f=0.020, equivalent to the pipe
line ABC b) the length of an 8cm diameter (f=0.015) pipe equivalent to the pipe
line ABC.
13) A horizontal pipe line carrying water at 0.03 m
3
/s reduces abruptly from 15cm
to 10cm diameter. Taking contraction coefficient C
C
=0.60. Determine the
pressure loss across the contraction. How this pressure loss compares with the
loss that would result if the flow direction is reversed?
14) Two pipes of diameter 40cm and 20cm are 300m each in length. When the
pipes are connected in series and the discharge through the pipe line is 0.1m
3
/s ,
find the loss of head incurred. What would be the loss of head incurred. What
would be the loss of head in the system to pass the same total discharge when
the pipes are connected in parallel, take f=0.03 for each pipe.
(P.S: FOR ANSWERS TO THE ABOVE PROBLEMS, DOWNLOAD THE
CONTENTS OF SESSION-9, VTU.AC.IN (E-LEARNING).
1 2
V1
V
entry
V
Fitting collar
V
entry
Fig.13
Fig 14.(a)
Fig 14.(b)
Fig 14. (c)
Fig 15
V2
exit
exit
V1
V2
flow
V1
V2
flow
Z1+p1/ Z2+p2/
Fig 17
Fig 16

1
1
2
2
1
1
2
2
Q
P2
FLOW MEASUREMENTS
Flow Through Orifices
An orifice is an opening of any cross section, at the bottom or on the side walls of
a container or vessel, through which the fluid is discharged.
If the geometric characteristics of the orifice plus the properties of the fluid are
known, then the orifice can be used to measure the flow rates.
Classification of orifices
Based on shape: circular, triangular, rectangular
Based on size :Small orifice (when the head over the orifice is more than five times its size I.e. H>5d,
Large orifice
Based on shape of the u/s edge :Sharp edge, Bell mouth
Based on flow: Free, Submerged
Flow through an orifice
As the fluid passes through the orifice under a head H, the stream lines converge
and therefore the jet contracts. The stream lines which converge are mostly
those from near the walls and they do so because stream lines cannot make right
angled bend in motion. This phenomenon occurs just down stream of the orifice,
and such a section where the area of cross section of the jet is minimum is know
as VENA CONTRACTA.
The pressure at Vena Contracta is assumed to be atmospheric and the velocity
is assumed to be the same across the section since the stream lines will be
parallel and equally spaced.
Downstream of Vena contracta the jet expands and bends down.
Figure(18) shows the details of free flow through a vertical orifice.
Applying Bernoulli's equation between (B) & (C) with the horizontal through BC
as datum and neglecting losses (h
L
)
L
h
g
V p
Z
g
V p
Z + + + + +
2 2
2
2 2
2
2
1 1
1

;
2 1
Z Z
,
1
H
p

V V V
2 1
, 0
0
2
0 0 0 0
2
+ + + + +
g
V
H
) 1 ( 2 gH orV Theoretical velocity
Velocity V in Eq(1) is known as TORRICELLIS VELOCITY.
Hydraulic Coefficients of an orifice
i)Coefficient of discharge (C
d
): It is defined as the ratio of actual discharge (Q
act
)
to the theoretical discharge (Q
th
).

,
_


th
act
d
Q
Q
C .
Value of C
d
varies in the range of 0.61 to 0.65
ii) Coefficient of Velocity (C
v
): It is defined as the ratio of actual velocity (V
act
) to
the theoretical velocity (V
th
).

,
_


th
act
V
V
V
C Value of C
v
varies in the range of
0.95 to 0.99
Coefficient of Contraction (C
c
): It is defined as the ratio of the area of cross
section of the jet at Vena of cross section of the jet at Vena Contracta (a
c
) to the
area of the orifice (a).

,
_


a
a
C
c
C
Value of C
c
will be generally more than 0.62.
Relationship between the Hydraulic Coefficients of an orifice
From continuity equation
Actual discharge Q
act
= a
c
x V
act
Theoretical discharge Q
th
= a x V
th
th
act c
th
act
V
V
x
a
a
Q
Q

or
c c d
xC C C
Equation for energy loss through an orifice
Applying Bernoullis equation between the liquid surface (A) and the centre of jet
and Vena Contracta (C) and considering losses (h
L
).
L
C C
C
A A
A
h
g
V p
Z
g
V p
Z + + + + +
2 2
2 2

, H Z
A
) ( 0 atmosphere p p
B A

, 0
A
V ) ( 0 city actualvelo p p
B A

L
h
g
Va
H + + + + +
2
0 0 0 0
2
)
2
(
2
g
Va
H h
L

gH C ButV
V a
2
Torricellis equation
) (
2
V L
HxC H h
) 1 (
2
V L
C H h
Equation for Coefficient of Velocity (C
V
) (Trajectory method)
Consider a point P on the centre line of the jet, such that its horizontal and
vertical coordinates are x and y respectively.
By definition, velocity
t
x
V
a
or,
a
V
x
t
Since, the jet falls through a vertical distance y under the action of gravity during
this time (t)
2
2
gt
y or
) 2 (
2
2
1

,
_

g
y
t
Equating equations (1) & (2)
2
1
2

,
_

g
y
V
x
a
But ,
gH C V
V a
2
2
1
2
2

,
_

g
y
gH C
x
V
2
1
2
1
2
1
2
1
2
1
2
1
2 2 y
g
x
H g
x
C
V

Hy
x
C
V
2

or
1
1
]
1

yH
x
C
V
4
2
Problems
1. The head of water over the centre of an orifice 30mm diameter is 1.5m. If the
coefficient of discharge for the orifice is 0.613, Calculate the actual discharge.
Solution: d=30mm = 3x10
-3
H=1.5m
C
d
=0.613
;
th
act
d
Q
Q
C

th d act
xQ C Q
gH xa C
d
2
( ) 2
1
3
5 . 1 81 . 9 2
4
) 10 30 (
613 . 0 x x x
x
x x


s m x Q
act
/ 10 35 . 2
3 3
or
lps Q
act
35 . 2
2. Compensation water is to be discharge by two circular orifices under a
constant head of 1.0m, measured from the centre of the orifices. What diameter
will be required to give a discharge of 20x10
3
m
3
per day? Assume C
d
for each
notch as 0.615.
Solution: d=?H=1m. Q
total
= 20x10
3
m
3
/day C
d
=0.615.
60 60 24
1
10 20
2
1
3
x x
x x x Q
act
s m / 1157 . 0
3

we know gH a C Q
d act
2
1 81 . 9 2
4
615 . 0 1157 . 0
2
x x x
xd
x

mm m d 5 . 232 2325 . 0
3. A jet of water issuing from an orifice 25mm diameter under a constant head of
1.5m falls 0.915m vertically before it strikes the ground at a distance of 2.288m
measured horizontally from the Vena Contracta. The discharge was found to be
102lpm. Determine the hydraulics coefficients of the orifice and the head due to
resistance.
Solution: d=25mm=25x10
-3
H=1.5m, y=0.915m, x=2.288m
Q
act
=102lpm = 102/60 = 1.7lps = 1.7x10
-3
m
3
/sec, C
d
=?, C
c
=?, h
L
=?
976 . 0
5 . 1 915 . 0 4
288 . 2
4
2 2

x x yH
x
C
V
( )
638 . 0
5 . 1 81 . 9 2 10 25
4 10 7 . 1
2
3
3

'

x x x x x
x x
Q
Q
C
th
act
d

999 . 0
976 . 0
638 . 0

,
_


v
d
C V C d
C
C
C xC C C
( )
2
1
v L
C H headlossh
( )
2
976 . 0 1 5 . 1 mm m h
L
2 . 71 0712 . 0
4. The head of water over a 100mm diameter orifice is 5m. The water coming out
of the orifice is collected in a circular tank 2m diameter. The time taken to collect
45cm of water is measured as 30secs. Also the coordinates of the jet at a point
from Vena Contract are 100cm horizontal and 5.2cm vertical. Calculate the
hydraulic coefficients of the orifice.
Solution:
D=100mm=0.1m, H=5m
Q
act
= Area of collecting tankxheight of water collected / time
s m x
x
/ 0471 . 0
30
45 . 0
4
2
3
2


X=100cm = 1m, y=5.2cm = 0.052m
C
d
=?, C
v
=?, C
c
=?
98 . 0
5 052 . 0 4
1
4
2 2

,
_

,
_

x x yH
x
C
v
605 . 0
5 81 . 9 2 1 . 0
4 0471 . 0
2

'

,
_


x x x
x
Q
Q
C
th
act
d

618 . 0
98 . 0
605 . 0

V
d
C
C
C
C
5. The coordinates of a point on the jet issuing from a vertical orifice are 0.4m &
0.003m. Neglecting air resistance, determine the velocity of the jet and the height
of water above the orifice in the tank.
Solution.
X=0.4m, y=0.3m, V=? H=?
Assume
We know
yH
x
C
V
4
2

,
_

2
2
2
2
2 2
1 03 . 0 4
4 . 0
4
4
x x yxG
x
H
x yHxG
H=1.33m
s m x x x gH G V / 115 . 5 33 . 1 81 . 9 2 1 2
6. A vertical orifice is fitted 0.2m above the bottom of a tank containing water to a
depth of 2m. If G=0.98. What is the vertical distance from the orifice of a point on
the jet 0.6m away from the Vena Contracta?
Solution
Head over the orifice H=(2-0.2)=1.8m
C
V
=0.98, y=?, x=0.6m
mm m
x x
y
xyx
or
yH
x
C
V
52 052 . 0
98 . 0 8 . 1 4
6 . 0
8 . 1 4
6 . 0
) 98 . 0 ( ,
4
2
2
2
2
2

,
_

7. A closed tank contains water to a height of 2m above a sharp edged orifice


1.5cm diameter, made in the bottom of the tank. If the discharge through the
orifice is to be 4lps. Workout the pressure at which air should be pumped into the
tank above water. Take C
d
=0.6.
Solution: (fig.19)
Q=4lps = 4x10
-3
m
3
/s
D=1.5x10
-2
m, C
d
=0.6
P
A
=?
3 3 3
/ 10 772 . 11 / 772 . 11 m kN x m N
air


Total head over the orifice

,
_

A
p
h H
gH a C Q
d act
2
( )

,
_

3
2
2
3
10 772 . 11
2 81 . 9 2
4
10 5 . 1
6 . 0 10 4
x
P
x x x
x
x x x
A

) ( / 83 . 0
2
Gauge m kN P
A

8. A closed tank contains 3m depth of water and an air space at 15kpa pressure.
A 5cm diameter orifice at the bottom of the tank discharge water to the tank B
containing pressurized air at 25kpa. If C
d
= 0.61 for the orifice. Calculate the
discharge of water from tank A.
Solution: fig(20)
d=5cm = 5x10
-2
m C
d
=0.61.
Total head over the orifice
( )

'

+

'

+
81 . 9
25 15
3

B A
p p
h H
H=1.9806m
9806 . 1 81 . 9 2
4
05 . 0
61 . 0 2
2
x x x
x
x gH a C Q
d act


lps s m x Q
act
47 . 7 / 10 47 . 7
3 3


9. A tank has two identical orifices in one of its vertical sides. The upper orifice is
4m below the water surface and the lower one 6m below the water surface. If the
value of C
v
for each orifice is 0.98, find the point of intersection of the two jets.
Solution.
yH
x
C
V
4
2

Given C
v
is same for both the orifices
2 2
2
2
1 1
2
1
4 4 H y
x
H y
x

) (
4 4
2 1
2 2
2
1 1
2
1
x x
H y
x
H y
x

) 1 ( 5 . 1 6 4
2 1 2 1
y ory y y
from figure(21)
( )
) 2 ( 2
4 6
2 1
2 1
+
+
y y
y y
Substituting eq(1) in eq(2) and simplifying
m y
y
y y
4
2 5 . 0
2 5 . 1
2
2
2 2

+
Again gives
H y
x
C
V
2 2
2
2
4

m x
x x
x
6 . 9
6 4 4
98 . 0
2
2
2

(points of intersection of the jets from the Vena contracts)


10. Two orifices have been provided in the side of the tank, one near the bottom
and the other near the top. Show that the jets from these two orifices will
intersect a plane through the base at the same distance from the tank if the head
on the upper orifice is equal to the height of the lower orifice above the base.
Assume C
v
to be the same for both the orifices.
Solution.
To show that x
1
=x
2
when H
1
=y
2
from figure(22)
y
1
=[y
2
+(H
2
-H
1
)---(1)
2 2
2
2
1 1
2
1
1
4 4
,
2
H y
x
H y
x
C C Given
V
V

or
2 2 1 1
4 4 H y H y
] ) ( [
2 2 1 1 2 2
H y H H H y +
2 2
2
1 2 1 1 2
H y H H H H y +
0 ) (
1 2 2 2 1
2
1
+ H H y H H H
substituting

0 0
0 ) ( ;
2 2 2 2 2
2
2 2 1

+ y H y y H y y H
11. A 4cm dia orifice in the vertical side of a tank discharges water. The water
surface in the tank is at a constant level of 2m above the centre of the orifice. If
the head loss in the orifice is 0.2m and coefficient of contraction can be assumed
to be 0.63. Calculate (I) the values of coefficient of velocity & coefficient of
discharge, (ii) Discharge through the orifice and (iii) Location of the point of
impact of the jet on the horizontal plane located 0.5m below the centre of the
orifice.
Solution
2 2 9.81 2 V gH x x
6.264 / V m s Head loss
2
2
L
Va
h H
g
_


,
2
0.2 2
2 9.81
Va
x
_


,
5.943 /
a
V m s
Coefficient of Velocity
5.943
0.943
6.246
a
v
V
C
V

Coefficient of discharge
d v C
C C xC 0.949 0.63 x 0.598
d
C
(ii) Discharge through the orifice
2
act d
Q C a gH
2
0.598 0.04 2 9.81 2
4
x x x x x

3
4.707 10 / 4.71 x m s lps


(iii) Coefficient of velocity
2
4
v
x
C
yH
or
2
4
V
yHC x
2
4 0.5 2(0.949) x x x 1.898 x m
12. An orifice has to be placed in the side of a tank so that the jet will be at a
maximum horizontal distance at the level of its base. If the depth of the liquid int
the tank is D, what is the position of the orifice? Show that the jets from the two
orifices in the side of the tank will intersect at the level of the base if the head on
the on the upper orifice is equal to the height of the orifice above the base.
Solution: fig(23)
By definition, Velocity V=x/t
Vt
But
2 V gH and
2
1
2
y gt
2
1
( )
2 2
x
D H g
gH
_



,
or
( )
2
4 x H D H
or
4 ( ) x H D H
For x to be maximum
0
dx
dH

4( 2 ) 0 D H
/ 2 H D
We know, x=Vt,
1
2 V gH
2
2
1
2
y H gt +
2
1
1
2 2
x
g
gH
1

1
1
]
( )
2
1 2
4 (1) x H y H +
( )
2
1
4 / 2 (2) x H y L +
Equating (1) & (2)
1 1 2 1 2 2
4 4 4 4 H y H H H H yH + +
1 2 1 2
4 4 H y yH H H
13. Two tanks with orifices in the same vertical plane are shown in figure.
What should be the spacing x for the jets to intersect in the plane of the base?
Assume C
V
=0.98 for each orifices. x=?
Solution: fig(25)
2 2
1 2
1 1 2 2
(1)
4 4
x x
y H y H

Assuming the coefficient of velocity C
V
to be the same for both the orifices, we
have (CV
1
) = (CV
2
)
where
1 1
2 2 0.4 1.6 y H m
2 2
2 2 1.6 0.4 y H m
1 2
0.4 , 1.6 H m H m
2 2
1 2
4 1.6 0.4 4 0.4 1.6
x x
x x x x

2
1
1
1 1
4
V
x
C
y H


2
1
0.98
4 1.6 0.4
x
x x

1 2
1.568 x m x
[ ]
1 2
2 1.568 3.136 x x x x m +
14. A large tank has a circular sharp edged orifice 25mm diameter in the
vertical side. The water level in the tank is 0.6m above the centre of the orifice.
The diameter of the jet at Vena contracta is measured as 20mm. The water of
the jet is collected in a tank 1.2m long x 0.6m wide and the water level rised from
0.15 to 0.75 in 7 minutes. Calculate the orifice coefficients.
Solution: Dia of jet at vena contracta d
c
=20mm
Dia of orifice = d = 25mm
Head over the orifice H = 0.6m
Depth of water collected in the measuring tank h=(0.75-0.15)=0.6m
Depth of water collected in the measuring tank A = 1.2x0.6 = 0.72m
2
Time taken for collecting 0.6m of water t=7min=7x60=420sec
Therefore actual discharge Q
act
= Area of measuring tank x depth of water
collected / time taken

0.72 0.6
. .
420
act
Ah x
i e Q
t
_


,
3 3
1.0286 10 / x m s

2
th
Q a gH
( )
2
3
25 10
2 9.81 0.6
4
x
x x x x

3 2
1.684 10 /
th
Q x m s

Coefficient of discharge
act
d
th
Q
C
Q

3
3
1.0286 10
1.684 10
x
x


,

0.61
d
C
Coefficient of contraction
C
c
= area of jet at vena contracta / area of orifice
2
2
4
4
C
c
d
C x
d

2
2
20
0.64
25
c
d
d
_
_


,
,
We know, C
d
=C
c
xC
V
Therefore, Coefficient of velocity
0.61
0.64
d
V
c
C
C
C


0.953
V
C
Coefficient of resistance
2
1
C
V
C
C
_


,
2
1
1
0.953
_


,
0.1008
r
C
15. A jet of water issuing from a vertical orifice in a tank under a constant
head of 4m. If the depth of water in the tank is 12m, at what depth another orifice
to be mounted vertically below the former one, so that both the jets meet at a
common point on the horizontal at the bottom of the tank? Assume C
v
to be the
same for both the orifices = 0.98.
Solution: fig(26)
2 2
1 2
1 2
1 1 2 2
,
4 4
V V
x x
C C
y H y H

From figure, (y
1
-y
2
)=(H
2
-H
1
)
( ) ( )
1 1
12 4 8 y y H m
( ) ( )
2 2 2
12 y y H H m
Equating the values of C
V
2 2
1 2
1 1 2 2
4 4
x x
y H y H

1 1 2 2
4 4 y H y H
1 2
x x Q
1 1 2 2
y H y H
( )
2 2
8 4 12 x H H
2
2 2
12 32 0 H H +
2
2
12 12 4 1 32
2 1
x x
H
x
+ t

8 4 mor m
H
2
= 8m is the correct answer.
Hence, the second orifice should be 4m below the first orifice.
16.
Water is to be discharged by two circular orifices under a constant head of 1m
above their centres. What should be the diameter of the orifices to give a
discharge of 20Mlpd? Assume a coefficient of discharge of 0.62.
Solution.
Total discharge=20Mlpd
(million litres per day)
6
20 10
231.48
24 60 60
x
lps
x x

Therefore Discharge per orifices
231.48
115.74
2
Q lps
2
0.11574 / orQ m s
But,
2
d
Q C a gH
0.11574
0.62 2 9.81 1
a
x x x

' ;

2
0.04214m
1
2
2
4 0.04214
4
d x
a d

_


,
=0.2316m
Therefore, Diameter of each orifice d = 231.6mm
17. What is the discharge through the 60mm diameter orifice shown in figure,
assuming the oil level remains constant
Solution. Fig(27)
Head of the orifice H
100
2
0.9 9.81 x
_
+

,
13.326 H mofoil
2
d
Q C a gH
( )
3
0.65 60 10 2 9.81 13.326
4
x x x x x x

3
0.02972 / m s
29.72lps
18. What is the discharge through a sharp edged slot 0.2 long x 10mm wide
at the bottom of a tank 0.5m diameter with 3m depth of water constant?
Solution.
Head over the orifice H=(2-0.2)=1.8m
0.98, ?, 0.6
V
C y x m
2
4
V
x
C
yH

2
2
0.6
(0.98)
4 1.8 xyx
_


,
0.052 52 y m mm
19. A vertical orifice is fitted 0.2m above the bottom of a tank containing
water to a depth of 2m. If C
V
=0.98. What is the vertical distance from the orifice
of a point on the jet 0.6m away from the Vena contracta?
Solution.
10
0.61 0.2 2 9.81 3
100
x x x x x
_


,
3 2
9.36 10 / x m s

9.36 Q lps

20. The coordinates of a point on the jet issuing from a vertical orifice are
0.4m & 0.3m. Neglecting air resistances, determine the velocity of the jet and the
height of water above the orifice in the tank. Assume C
V
=0.98.
Solution.
X=0.4m, y=0.3m, V=?, H=?
2
4
V
x
C
yH

2
0.4
0.98
4 0.3 x xH

0.1388 H m
2 2 9.81 0.1388 1.65 / V gH x x m s
Mouth Pieces
A mouth piece is a short tube or pipe connected in extension with an orifice
Classification of Mouth Pieces
Depending on the position with respect to the tank: External, Internal
Depending on shape :Cylindrical,Convergent, Divergent
Nature of flow: Running Full,Running Free
External Cylindrical Mouthpiece fig(28)
It is a short pipe whose length is two or three times the diameter.
H=Head over the centre of the mouth piece
V
O
=Velocity of the liquid at Vena Contracta
a
c
=Area of flow at Vena Contracta
V
1
=Velocity of liquid at outlet
a
1
=Area of mouth piece at outlet.
C
c
=coefficient of contraction
Applying continuity equation between & (1) &(1)
a
c
c
c
=a
1
v
1
1
1 c
c
a
V V
a

1
0.62
c
ac
c coefficientofcontraction
a

1
1
(1)
0.62
c
V V
As the jet flows from to (1) (1) there will be loss of head due to sudden
enlargement of flow, and this value can be calculated from the relation.
( )
2
1
2
1
1
0.62
2 2
C
L
V
V
V V
h
g g
_

,

2
1
0.375
(2)
2
L
h V
g

Applying Bernoullis equation between (A) and (1) (1) with the centre line of the
mouth piece as datum and considering head loss h
L
due to sudden expansion.
2 2
1 1
1
2 2
A A
A L
p V p V
Z Z h
g g
+ + + + +
1
1
0, , 0( )
A
p pA
Z Z H atmosphericpressure


0( )
A
V Negligible
2 2
1 1
0.375
0 0 0 0
2 2
V V
H
g g
+ + + + +
2
1
1.375
2
H V
g

or
1
2
1.375
gH
V
1
0.853 2 (3) V gH
By definition, Coefficient of velocity
C
V
=Actual velocity/Theoretical velocity
0.853 2
. ,
2
V
gH
i e C
gH

0.853
V
C
At the exit of the mouth piece C
C
=1
1 0.853 0.853
d c v
C C xC x
Hence, for an external cylindrical mouth piece C
d
=(=0.853) is more than that of
an orifice.
Pressure head at Vena contracta
Applying Bernoullis equation between (A) & with the centre line of the mouth
piece as datum & neglecting losses.
2 2
2 2
c c A A
A c L
p V p V
Z Z h
g g
+ + + + +
, 0, 0, 0, ?
c
A A C L
P pA
H V Z Z h


2
0 0 0 0
2
c c
p V
H
g
+ + + + +
2
2
c c
p V
H
g

But,
2
1 1
1.375 ,
2 0.62
C
V V
H V
g

1
2
0.853 2
1.375
gH
V gH
0.853 2
&
0.62
C
gH
V
2
0.853 2 1
0.62 2
C
gH p
H x
g

_


' ;

,

1.893
C
p
H H


0.893
C
p
H


Negative sign indicates that the pressure at the Vena contracta is less than
atmospheric pressure or the pressure is negative
Problems
1. Find the discharge from a 80mm diameter external mouthpiece, fitted to a side
of a large vessel if the head over the mouth piece is 6m.
Solution.
( )
2
3
80 80 10
4
d mm a x x



3 2
5.026 10 x m

2
d
Q C a gH
For a cylindrical mouth piece C
d
=0.853
3
0.853 5.026 10 2 9.81 6 Q x x x x x


3
0.04652 / m s
46.52 Q lps
2. An external cylindrical mouthpiece of 100mm diameter is discharging water
under a constant head of 8m. Determine the discharge and absolute pressure
head of water at Vena contracta. Take C
d
=0.855 and C
C
for Vena contracta
=0.62. Take atmospheric pressure head =10.3m of water
Solution.
H=8m, Q=?, C
d
=0.855, C
c
=0.62
2
3 2
0.1
100 0.1 7.854 10
4
x
d mm m x m



10.3
a
p
mofwater


3
2 0.855 7.854 10 2 9.81 8
d
Q C a gH x x x x x


3
0.08413 / sec 84.13 Q m lps
We know
0.893 ( 0.62)
c
c
p
H whenC


0.893 8 7.144 ( )
c
p
x mofwater Gauge


c c c
p p p
absolute gauge

_ _ _

' ;
, , ,
(10.3 7.144)

3.156 ( ) m Abs
3. An external cylindrical mouth piece 60mm diameter fitted in the side of a tank
discharges under a constant head of 3m, for which C
V
=0.82
Determine i) the discharge in lps
ii) absolute pressure at Vena contracta
iii) Maximum head for steady.
Flow assuming that separation occurs at 2.5m of water absolute. Local
barometer reads 760mm H
g
.
Solution.
(i)Discharge(Q)
At the exit of the mouth piece C
C
=1
1 0.82
d c v
C C xC x

2
d
Q C a gH
( )
2
3
60 10
0.82 2 9.81 3
4
x x
x x x x

' ;


3
0.0178 / 17.8 m s lps
(ii) Absolute Pressure head at Vena contracta
Applying Bernoullis equation between (A) & with the centre line of the mouth
piece as datum and neglecting losses h
L
2 2
1
2 2
C C A A
A L
p V p V
Z Z h
g g
+ + + + +
2
0 0} {0 0
2
c C
p V
H
g
+ + + + +
2
(1)
2
C C
p V
H
g
+
From Continuity equation Q=aV
Q
V
a

( )
2
3
0.0178
0.62 60 10
4
C
C
Q
V
C xa
x x x





' ;


10.154 /
c
V m s

(1) FromEq

2
2
c c
p V
H
g

2
10.154
3
2 9.81 x
_


,
=2.255m of water (Gauge)
c atim c
Gauge
p p p

_ _


, ,
760
atm
p
mmofmercury



1 1 2 2
10.336 ( ) mofwater S H S H Q
( ) 10.336 2.555 7.781( )
c
abs
p
Absolute

_


,
(iii) H
max
for steady flow
Applying Bernoullis equation between Vena contracta and exit of the mouth
piece with the centre line of the mouth piece as datum & considering head loss
h
L
due to sudden expansion of flow.
2 2
1 1
1
2 2
c C
C L
p V p V
Z Z h
g g
+ + + + +
2
min 0.62, 0.375
2
C L
V
Assu gC h
g

( )
2
2 2
/ 0.62
0.375
0 7.836 0 0
19 62 2 2
V
V V
g g
+ + + +

2
7.836 1.2265
2
V
g

2
7.836
6.389
2 1.226
V
mofwater
g
_


,
2
V
weknowV C gH
2
2
1
2
V
V
H x
g C

max 2
1
6.389
0.82
H x
9.5mofwater
Alternatively,
0.82
c a
p p
H


-7.781=0-0.82xH
max
H
max
=9.49m of water
Submerged Orifice
A fully submerged orifice is one in which the entire outlet side or the downstream
side is completely under the liquid. It is also known as a drowned orifice.
Consider points (1) and (2) situated upstream of orifice and at the Vena contracta
respectively.
H1=Height of water above the top of the orifice on the upstream side.
H2=Height of water above the bottom level of the orifice.
H=Difference in water level
b=width of orifice
C
a
=Coefficient of discharge.
Height of water above the centre of orifice on upstream side
2 1 1 2
1
2 2
H H H H
H
_
+
' ;

,
Height of water above the centre of the orifice on the downstream side
1 2
2
H H
H
_


,
Applying Bernoullis equation between (1) and (2) with the horizontal passing
through (A) & (B) as datum and neglecting losses. (h
L
)
{
2 2
1 1 2 2
1 2
2 2
L
p V p V
Z Z h
g g
+ + + + +
1 1 2 2 1 2
1 2
0, ;
2 2
p H H p H H
Z Z H

+ + _ _


, ,
1
0 V
(negligible)
2
1 2 1 2 2
0 0 0 0
2 2 2
H H H H V
H
g
+ +
+ + + + +
2
2
2
V
or H
g

2
2 V gH
From continuity equation
Discharge Q
act
=C
d
xarea of orifice x velocity
( )
2 1
2
act d
Q C xb H H x gH
Large Rectangular Orifice
An orifice is said to be large when the head acting on it is five times the depth of
the orifice. Unlike in the case of a small orifice, the discharge cannot be
calculated from the equation
for the reason that the velocity is not constant over the entire cross section of the
jet.
Consider an elementary horizontal strip of depth dh at a depth h below the free
surface of the liquid as shown.
Area of the strip
Theoretical velocity through the strip 2gh
Discharge through the elementary strip 2 dQ bxdhx gh
Therefore through the entire orifice is obtained by integrating the above equation
between the limits H
1
= and H
2
.
2
1
. , 2
H
d
H
i e Q C xbx ghdh

1 2
2
1
2
H
d
H
C b g h dh

3 3
2 2
2 1
2
2
3
d
Q C b gx H H
1

1
]
Problems
1. Find the discharge through a fully submerged orifice of width 2m if the
difference of water levels on both the sides of the orifice be 800mm. The height
of water from the top and bottom of the orifice are 2.5m and 3m respectively.
Take C
d
=0.6
Solution.
For a submerged orifice.
( )
2 1
2
d
Q C b H H x gH
Where, C
d
=0.6, b=2m, H
2
=3m, H
1
=2.5m, H=800mm = 0.8m
( ) 0.6 2 3 2.5 2 9.81 0.8 Q x x x x x
3
2.377 / Q m s
2. Find the discharge through a totally drowned orifice 1.5m wide and 1m deep, if
the difference of water levels on both the sides of the orifice is 2.5m, Take
C
d
=0.62.
Solution.
b=1.5m, d=1m, H=2.5m, C
d
=0.62
2
d
Q C a gH
0.62 1.5 1 2 9.81 2.5 x x x x x
3
6.513 / Q m s
3. Find the discharge through a rectangular orifice 3m wide and 2m deep fitted to
a water tank. The water level in the tank is 4m above the top edge of the orifice.
Take C
d
=0.62.
Solution.
For a rectangular orifice
3 3
2 2
2 1
2
2
3
d
Q C b g H H
_


,
where
B=3m, C
d
=0.62, H
2
=(4+2)=6m, H
1
=4m.
3 3
2 2
2
0.62 3 2 9.81 6 4
3
Q x x x x x
_


,
4. A rectangular orifice 1m wide and 1.5m deep is discharging water from a
vessel. The top edge of the orifice is 0.8m below the water surface in the vessel.
Calculate the discharge through the orifice if C
d
=0.6. Also calculate the
percentage error if the orifice is treated as small.
Solution.
For a rectangular orifice.
3 3
2 2
2 1
2
2
3
d
Q C b g H H
_


,
3 3
2 2
2
0.6 1 2 9.81 2.3 0.8
3
x x x x x
_


,
3
arg
4.912 /
l e
Q m s
2
small d
Q C a gH
1.5
0.6 1 1.5 2 9.81 0.8
2
x x x x x
_
+

,
3
4.963 / sec
small
Q m
arg
arg
% 100
small l e
l e
Q Q
Error x
Q

4.963 4912
100
4.912
x
_


,

% error=1.04 %
Flow Over Notches & Weirs
A notch is an opening made in the side wall of a tank such that the liquid surface
in the tank is below the upper edge of the opening. Generally notches are made
of metallic plates and their use is limited to laboratory channels.
A weir is a masonry/concrete structures built across an open channel so as to
rise the water level on the upstream jside and to allow the excess water to flow
over the entire length onto the downstream side.
Classification
a) Depending on shape:
i) Rectangular
ii) Triangular
iii) Trapezoidal
b) Depending on the shape of the crest
i) Sharp crested
ii) Broad crested.
c) Depending on flow
i) Free
ii) Submerged
d) Depending on Ventilation
i) Fully aerated
ii) Depressed
iii) Clinging or Drowned.
Definition sketch of a Notch.(fig.31)

a)Flow over a Triangular Notch.
Figure(32) shows the details of flow over a V notch.
2=Central angle
H=Head over the notch
Consider an elemental strip of thickness dh at a depth h below the free
surface as shown. Discharge through the strip dq=area x velocity
2 2 dq xdhx gh
Discharge over the entire notch
0 0
2 2
Q H
dq xdhx gh

1
2
0
2 2
H
Q g xh dh

In the above equation x can be eliminated.Interms of & H. In otherwords


tan
( )
x
H h

or

( ) tan x H h
( ) ( )
1
2
0
2 2 tan
H
Q g H h h dh

1 3
2 2
0
2 2 tan
H
g Hh h dh
_


,

integrating
3 5
2 2
0
2 2
2 2 tan
3 5
H
Q g Hh h


' ;

5
2
2 2
2 2 tan
3 5
g H


' ;

( )
5
2
5 3
4 2 tan
15
g H

5
2
8
2 tan (1)
15
Q gH
5
2
8
2 tan
15
act d
Q C gH
d
C
=Coefficient of discharge
In deriving Eq(1), velocity of approach V
a
is neglected. If the head due to this
velocity of approach is considered,
then, ( )
5
5
2
2
8
2 tan (3)
15
act d a
Q C g H h ha

+
' ;

Problems
1. A triangular notch discharges 0.0110m
3
/s under a head of 0.2m. Find the
angle of the notch, if C
d
=0.626.
Solution.
3
0.0113 / , 0.2
0.626
d
Q m s H m
C

5
2
8
2 tan
15
d
Q C gH
( )
5
2
15 0.0110
tan
8
0.626 2 9.81 0.2
x
x x x

' ;


0
22.6
Therefore angle of the notch
0
2 45.2
2. A right angled triangular notch discharges 0.143m
3
/s. Find the head over the
notch if C
d
=0.6.
Solution.
0 0
3
2 90 , 45
0.143 / , ?
0.6
d
Q m s H
C

5
2
8
2 tan
15
d
Q C gH
0
15 0.143
8 0.6 19.62 tan 45
H x
x x

' ;

0.3995 399.5 H mm
3. 150lpm of water is expected to flow down an irrigation furrow. Design the
weir, if a minimum head of 100mm is desired. Assume C
d
=0.61.
Solution:
3 3
150
150 2.5 2.5 10 /
60
Q lpm lps x m s


100 0.1 , 0.61
d
H mm m C

5
2
8
2 tan
15
d
Q C gH
( )
3
5
2
15 2.5 10
tan
8
0.61 19.62 0.1
x
x
x x

' ;


0
0.5486 28.75
0
2 57.5 60 centralangle
Hence, it is suggested to use a 60
0
V notch.
4. Calculate the top width and depth of a triangular notch capable of discharging
700lps. The weir discharges 5.7 lps when the head over the crest is 7.5cm. Take
C
d
=0.62.
Solution.
3
700 0.7 / , ? Q lps m s H
3 3
1 1
5.7 10 / , 0.075 , 0.62
d
Q x m s H m C


5
2
8
2 tan
15
d
Q C gH
5 5
2 2
1
8 15
2 tan 2 tan
15 8
d d
Q
C gH x C gH
Q

5
2
1
0.075
Q H
Q
_


,
5
2
3
0.7
5.7 10 0.075
H
x

_


,
0.514 H m
Also,
5
2
8
2 tan
15
d
Q C gH
or
( ) ( )
1 5
2 2
15 0.7
tan
8
0.62 2 9.81 0.514
x
x x x

' ;


0 0
2.523
68.38 2 136.76 or


Top width of the notch =
0
2 0.514 tan 68.38
2.594
x x
metres

Flow over a Rectangular Notch


Figure(33) shows the details of the flow over a rectangular notch.
L=length of the notch
H=head over the notch
Consider a small strip of thickness dh at a depth h below the liquid surface
Discharge through the strip d
q
=areaxvelocity
2 dq Lxdhx gh
Total discharge
1
2
0 0
2
Q H
dq L g h dh

3
2
0
2
2
3
H
Q L g h

3
2
2
2
3
Q L gH
3
2
2
2 (1)
3
act d
Q C gLH
C
d
=Coefficient of discharge, its average value is about 0.62.
End Contraction
When the length of the weir(L) is less than the width of the channel (B), the
nappe contracts at the sides, and this is knows as end contractions.(fig34)
According to Francis, the effective length of flow over the notch is given by
Substituting this value in EQ(1) and simplifying
( )
2 3
2 0.1 (2)
3 2
d
Q C g L nH H
A notch without end contraction is known as a suppressed notch.
Velocity of approach (V
a
)
The total head over the weir will be the sum of static head (H) and velocity head
(h
a
), velocity head
2
2
a
a
V
h
a
is due to the
velocity of the liquid approaching the notch.
On similar lines, considering a strip of uniform thickness dh at a depth h below
the liquid surface.
Discharge through the strip d
q
=area x velocity. ( ) 2
a
dQ Lxdhx g H h +
Therefore Total discharge is given by
( )
1
2
0 0
2
Q H
a
dq L g H h dh +

( )
3
3
2
2
2
2
3
L g H ha ha

+
' ;

( )
3
3
2
2
2
2
3
d a
Q C gL H h ha

+
' ;

Empirical Formula
(i)Francis Formula ( ) ( )
3
3
2
2
1.84 0.1 Q L nH H ha ha
1
+
1
]
(ii)
(ii)
(ii)Bazins formula
3
2
0.003
0.405 2 Q L gH
H
_
+

,
(iii)
(iii)
(iii)Rehbock formula
( )
( )
3
2
0.053 0.011
0.403 2 0.0011
H
Q L g H
Z
1 +
+ +
1
]
Considering velocity of approach and End contraction, we have
( ) ( )
3
3
2
2
2
2 0.1
3
d a
Q C g L nH H h ha

+
' ;

Considering velocity of approach and End contraction, we have
( ) ( )
3
3
2
2
2
2 0.1
3
d a
Q C g L nH H h ha

+
' ;


5.Find the discharge over a rectangular notch of crest length 400mm. When the
head of water over the crest is 50mm. Take C
d
=0.6.
Solution.
3
2
2
2
3
d
Q C gLH
( )
3
2
2
0.6 2 9.81 0.4 0.05
3
x x x x x
3 3
7.92 10 / 7.92 Q x m s lps


6. A rectangular weir 9m long is divided into 3 bays by two vertical post each
300mm wide. If the head of water over the weir is 500mm, Calculate the
discharge, given C
d
=0.62.
Solution.
( )
3
2
2
2 0.1
3
d
Q C g L nH H
n=number of end contractions=6.
L=clear length of weir=9-3x0.3=8.1
( )
3
2
2
0.62 2 9.81 8.1 0.1 6 0.5 0.5
3
Q x x x x x x
3
5.05 / Q m s
7. The discharge over a rectangular weir is 0.4m
3
/s when the head of water is
0.20m. What would be the discharge if the head is increase to 0.3m?
Solution.
Q
1
=0.4m
3
/s H
1
=0.20m
Q
2
=? H
2
=0.3m
3
2
2 2
1 1
Q H
Q H
_


,
3
2
3
2
0.3
, 0.4 0.735 /
0.20
or Q x m s
_


,
8. A rectangular channel 6m wide carries a flow of 1.5m
3
/s. A rectangular
sharp crested weir is to be installed near the end of the channel to create a depth
of 1m upstream of the weir. Calculate the necessary height . Assume C
d
=0.62.
Solution: fig(35)
L=6m, Q=1.5m
3
/s, C
d
=0.62.
Y=(Z+H)=1m
Velocity of approach
V
a
= Discharge/area of flow in the channel
1.5
0.25 /
1 6
a
V m s
x

Head due to velocity of approach
2
2
a
V
ha
g

2
3
0.25
3.185 10
2 9.81
a
h x m
x


But,
( )
3
3
2
2
2
2 ( )
3
d a
Q C gL H h ha

+
' ;

( ) ( )
3 3
3 3
2 2
2
1.5 0.62 2 9.81 6 3.185 10 3.185 10
3
x x x x x H x x


+
' ;

0.266 H m , And height of the weir
( ) 1 0.266
0.734 Z m


2
1
0.05097
2 9.81
a
h m
x

( )
3
3
2
2
2
2 ( )
3
d a
Q C gL H h ha

+
' ;

( ) ( )

'

+ 2
3
2
3
05097 . 0 05097 . 0 2 . 1 81 . 9 2 6 . 0
3
2
645 . 2 xLx x x x
9. A rectangular sharp crested weir is required to discharge 2.645m
3
/s of wate
under a head of 1.2m. If the coefficient of discharge is 0.6 and the velocity of
approach near the weir is 1m/s. Find the length of the weir.
Solution.
Q=2.645m
3
/s H=1.2m
C
d
=0.6 V
a
=1m/s L=?
Head due to velocity of approach
2
2
a
a
V
h
g

And height of the weir ( ) Z y H


( ) 1 0.266
0.734 Z m

10. A rectangular sharp crested weir is required to discharge 2.645m


3
/s of water
under a head of 1.2m. If the coefficient of discharge is 0.6 and the velocity of
approach near the weir is 1m/s. Find the length of the weir.
Solution.
Q=2.645m
3
/s H=1.2m
C
d
=0.6 V
a
=1m/s L=?
Head due to velocity of approach
2
2
a
a
V
h
g

2
1
0.05097
2 9.81
a
h m
x

( )
3
3
2
2
2
2 ( )
3
d a
Q C gL H h ha

+
' ;

( ) ( )
3 3
2 2
2
2.645 0.6 2 9.81 1.2 0.05097 0.05097
3
x x x xLx

+
' ;

1.076 107.6 L m cm
11. Water passing over a rectangular notch flow subsequently over a right
angled triangular notch. The length of the rectangular notch is 600mm and
C
d
=0.62. if the C
d
value for the V-notch is 0.60, what will be its washing head,
when the head on the rectangular notch is 20cm.
Solution.
Rectangular Notch.
L=600mm=0.6m
C
d
=0.62, H=0.2m
3
2
2
2
3
d
Q C gLH
( )
3
2
2
0.62 2 9.81 0.6 0.2
3
x x x x x
3
0.09825 / Q m s
0 0
2 90 , 45 , ?, 0.6
d
H C
Since the same discharge of 0.09825m
3
/s is passing over the V-
notch, we have
5
2
8
2 tan
15
d
Q C gH
( )
5
2
8
0.09825 0.6 2 9.81 tan
15
x x x x H
5
2
0.0693.
0.3438
H
H m


Types of Nappe
The equations derived for the discharge over notches were under the assumption
that pressure under the nappe is atmosphere. However, when the liquid if flowing
over the notch (suppressed), it touches the walls of the channel and the air gets
dissolved or entrained in water, continuation of this process results in a negative
pressure i.e. partial vaccum under the nappe. Finally the nappe gets deflected
closer to the weir wall.
The pressure on the inner side of the the nappe decides its type in the following
ways.
a)Free Nappe
In this type, the stream of water passing over the weir the springs clear of the
weir. (fig.36a)
b) Depressed Nappe
In this type, a partial vaccum is created between the nappe and the weir.
Discharge for such a flow situation is 8 to 10% greater than that with a free
nappe.(fig.36b)
c) Clinging Nappe
In this type the nappe totally adheres to the face of the weir. The discharge in this
case would be 20 to 30% more than that in a fully aerated nappe).(fig36c)
Ventilation of Weirs
The nappe emerging from a weir should be of a correct form, so that the
discharge equations derived for them are valid.
For accurate gauging of flow the nappe should spring clear or it should be free. In
other words the space between the weir and nappe should be maintained under
atmospheric conditions, particularly when the weir is suppressed.
In practice ventilation holes are made on the weir walls so that air circulates
freely between the weir and the nappe.
This is known as ventilation or ventilation of weirs.(fig36d).
Submerged Weir
A weir is said to be submerged when water level on both the upstream and
downstream sides are above the crest level of the weir as shown in figure (37).
H
1
and H
2
are the heads over the weir on the upstream and downstream sides.
In the case of submerged weir, it is necessary to derive the discharge equation
considering that the flow over the weir is a combination of a free weir and a
submerged orifice.
In other words, the flow Q
1
between H
1
and H
2
is considered as a free weir and
Q
2
between H
2
and the weir crest as a submerged orifice.
For a free weir ) 1 ( ) ( 2
3
2
2
3
2 1 1 1
H H g L Cd Q
For a submerged orifice. ) 2 ( ) ( 2
2 1 2 2 1
H H g LH Cd Q
) 3 (
2 1
+ Q Q Q
Cd
1
=0.58 and Cd
2
= 0.80 are usually considered for the weir and the orifice.
As in the earlier cases the head due to velocity of approach h
a
=Va
2
/2g can also
be considered.
In such a case ) 4 ( ) ( 2
3
2
2
3
2
3
2 1 1 1
+ ha ha H H g L Cd Q
) 5 ( ) ( 2
2 1 2 2 1
+ ha H H g LH Cd Q
In all the above equation L=length of the notch or weir.
Problems
1) A submerged weir 1m high spans the entire width of a rectangular channel 7m
wide. Estimate the discharge when the depth of water is 1.8m on the upstream
side and 1.25m on the downstream side of the weir. Assume C
d
=0.62 for the
weir.
Solution (fig.38).
Q
submerged weir
= Q
weir
+Q
submerged orifice
( ) ( )
2 1 2 2 1 1
2
2
3
2
3
2
H H g LH C H H x g L Cd
d
+
( ) ( ) 25 . 0 8 . 0 62 . 19 25 . 0 7 62 . 0
2
3
25 . 0 8 . 0 62 . 19 7 62 . 0
3
2
+ x x x x x x x x x
s m Q / 25 . 6
3

2) The upstream and downstream water surfaces are 150mm and 75m above the
crest of a drowned weir. If the length of the weir is 2.5m, find the discharge, the
coefficients of discharge for the free and drowned portions may be taken as 0.58
and 0.8 respectively. Allow for velocity of approach.
Solution.
H
1
=1500mm=15m, H
2
=75mm=0.075m
L=2.5m, Cd
1
=0.58, Cd
2
=0.8
( ) ( )

'

+
2 1 2 2
2
3
2 1 1
2 2
3
2
H H g LH Cd H H g xLx xCd Q
( )

'

+ ) 075 . 0 15 . 0 ( 62 . 19 075 . 0 5 . 2 8 . 0 075 . 0 15 . 0 62 . 19 5 . 2 58 . 0


3
2
2
3
x x x x x
s m Q / 2698 . 0
3

Velocity of approach
softheweir u areaofflow
Q
V
a
/

s m
x
/ 719 . 0
15 . 0 5 . 2
2698 . 0

'

m
x g
V
h
a
a
00264 . 0
81 . 9 2
719 . 0
2
2 2

( )

'

+ +
+

0264 . 0 ) 075 . 0 15 . 0 ( 62 . 19 075 . 0 5 . 2 8 . 0


0264 . 0 075 . 0 15 . 0 62 . 19 5 . 2 58 . 0
3
2
2
3
x x x
x x
Q
s m Q / 271 . 0
3

OGEE WEIR
When the weir is suppressed and its height is large, the nappe emerging out may
be subjected to the problems of ventilation. Hence, in such cases the weir profile
downstream is constructed conforming to the shape of the lower side of the
nappe. Such a weir is known as a spillway or ogee weir.
The cross section of an ogee weir will be shown(39). The coordinates of the
spillway profile can be worked out for the head H using the equation.
y H x
85 . 0 85 . 1
2
The u/s face of the spillway is generally kept vertical.
The discharge equation for an ogee weir will be
2
3
2
3
2
LH g C Q
d

Same as that for a suppressed rectangular notch.


Problem
1. Calculate the discharge over an ogee weir of 8.5m length, when the head
over the crest is 2.15m and C
d
=0.61.
Solution.
L=8.5m, H=2.15m, C
d
0.61
2
3
2
3
2
LH g C Q
d

2
3
) 15 . 2 ( 5 . 8 81 . 9 2 61 . 0
3
2
x x x x x s m Q / 27 . 48
3

Broad Crested Weir


A weir is said to broad crested when its width (parallel to flow) b is greater than
0.5xmaximum head acting on it (fig.40).
xH b e i 5 . 0 , . >
Let L=length of the weir
H=Head of water u/s of the weir w.r.t. the crest
h=Depth of water over the weir crest
V=Vel. Of flow over the weir
Applying Bernoullis equation between (1) and (2) with the crest of the weir as
datum & neglecting losses (h
L
)
g
V
h H
g
V p
Z
g
V p
Z
2
0 0 0
2
{ }
2
2
2
2 2
2
2
1 1
1
+ + + +
+ + + +

) 1 ( ) ( 2 h H g V
Discharge over the weir Q=area of flow over the weir x vel. of flow over the weir.
i.e. ) ( 2 ) ( h H g x hxL Q
Actual discharge ( ) ) 2 ( 2 h H g Lh C Q
d act
From Equation (2), we see that Q
act
is a function of h for a given value of H.
Q
is maximum when 0

dh
Q
( ) 0 2
1

'

h H h
h
( ) ( ) 0 2
1
2
1

'

h
h
h H h H
dh
hx
( ) ( ) ( ) 0 1
2
1
1
2
1
1
2
1

'

+
x
h H R H hx
0 ) (
) ( 2
2
1
2
1

'

h H
h H
h
( ) H h h H h 2 3 2
max
) (
3
2
orQ conditionf H orh
Substituting the value of h in eq(2) and simplifying.
( )

,
_


,
_

H H g x H L C Q
d act
3
2
2
3
2
max
3
2
3
2 H
g LHx C
d

( )
2
3
max
2
3 3
2
H g L C Q
d act

Problems
1. Determine the discharge over a broad crested weir 26m long, the upstream
level of water is measured as 0.5m above the crest level. The height of the weir
is 0.6m and the width of the approach channel is 36m. Take C
d
=0.9.
Solution.
For a broad crested weir. ( )
2
3
max
2
3 3
2
H g L C Q
d act

2
3
) 5 . 0 ( 62 . 19 26 9 . 0
3 3
2
x x x x
( ) s m Q
act
/ 105 . 14
3
max

Since, the width of the channel, we have to consider the velocity of approach V
a

,
_

hechannel Areaofflow
Q
V e i
a
int
, .
s m
x
/ 356 . 0
) 5 . 0 6 . 0 ( 36
105 . 14

,
_

+
3
2 2
10 466 . 6
81 . 9 2
356 . 0
2

x
x g
V
h
a
a
( ) ( ) 2
3
max
2
3 3
2
ha H g L C Q
d act
+
( )
2
3
3
10 466 . 6 5 . 0 62 . 19 26 9 . 0
3 3
2

+ x x x x x
( ) s m Q
act
/ 379 . 14
3
max

2. A reservoir discharges water at 60,000 m
3
/day over a broad crested weir, the
head of length of the weir, if C
d
=0.65.
Solution.
Q=60,000m
3
/day=60,000/24x60x60=0.694m
3
/s
H=500mm=0.5m C
d
=?, L=?
( )
2
3
max
2
3 3
2
H g L C Q
d act

B
A
y
C
R
x

'


2
3
) 5 . 0 ( 62 . 19 65 . 0 2
694 . 0 3 3
x x x
x
L
metres L 77 . 1
3. A channel of 45m
2
cross sectional area, discharging 50 cumecs of water is to
be provided with a broad crested weir. If the crest of the weir is 1.6m below the
upstream water surface, find the length of the weir, if C
d
=0.85.
Solution.
( ) ( ) 2
3
max
2
3 3
2
ha H g L C Q
d act
+
g
x
A
Q
g
V
whereha
a
2
1
2
2
2

,
_


m x 629 . 0
62 . 19
1
45
50
2

,
_

( ) 2
3
0629 . 0 6 . 1 62 . 19 85 . 0
3 3
2
50 + xLx x
L 1076 . 3 50
or m m L 1 . 16 089 . 16
Fig 18
Fig 19.
H
Vena contracta
Centreline of orifice
Air
Y2
H1=4m
O1
O2
Fig 21
Fig 22
h=2m
X1 = X2

Y
1
H2=6m
H1
Y1
X1=X2
Y2
H
X
Y
V
H3
y
H1
Y1
Fig 23
Fig 24
H2
Point of intersection of the jet
D
H2
X1=X2=X3
Y2
H1=0.6m
2m
Y1
H2=1.6m
Y2
X1 X2
Y=12m
H2=?
H1=4m
Y1
Y2
X1=X2
Fig 25
Fig 26
X=?
100 KN/m
2
Air
2m
Oil s=0.9
Fig 27
Fig 28
H
VENA CONTRACTA
MOUTH PIECE
H2
H1
H
1 2
H2
H
H1
h
b
d
dh
H
Flow Q
Apex or crest
Fig 29
Fig 30
Fig 31
Velocity of
approach Va
Channel Bed
Dead water
Z
Notch
Nappe
H
ha
dh
h
x

h
dh
H
L
Fig 32
Fig 33
Elemental Strip
B L
Flow
Notch
Side wall
y
H
Z
Nappe
Nappe
Fig 34
Fig 35
Fig 36. (a)
Fig 36. (b)
Fig 36 (c)
flow
Partially
Ventilated
Nappe
Ventilation holes
Q1
Q2 H2
H1
Weir
1.8m
H1=0.8m
1m
H2=0.25m
1.25m
Fig 36.(d)
Fig 37
Fig 38
Fig 39
Crest of ogee weir
H
v
h
flow
Channel bed
Fig 40
(Additional Problems)
1.Find the diameter of a sharp edged orifice sufficient to discharge 3x10
6
litres of
water per day. Under a constant head of 12m. Take C
d
=0.62
2.
2.A closed tank partially with water upto a height of 2m has an orifice of 2.75cm
diameter at its bottom. Determine the air pressure over the water surface for a
discharge of 5 lps, through the orifice, C
d
=0.62
Top surface of nappe
Concrete or masonry
Solid boundary coinciding with the
bottom surface of the nappe
weir
3. Water is discharged vertically upwards through a 5cm diameter orifice under a
pressure of 180N/cm
2
. C
V
for the orifice is 0.92. Determine the height to which
the jet will rise, when
i) there is no air friction
ii) when the friction produces a retardation of 2m/s
2
.
4. In performing an experiment to determine the different coefficients of a sharp
edged orifice, a jet of water issuing horizontally from the orifice of 2.5cm diameter
under a constant head of 1.5m fell through 0.9m vertically and struck the ground
at 2.3m horizontally from Vena contracta. The time required to discharge 91litres
of water was found to be 53 seconds. Find the values of C
d
, C
V
and C
C
for the
orifice.
5. A Jet of water issuing from a vertical orifice 0.025m diameter under a constant
head of 1.5m falls 0.915m vertically, before it strikes the ground at a distance of
2.288m, measured from Vena contracta horizontally. The discharge was found to
be 0.102 m
3
/min. Determine C
d
, C
V
, C
C
, & C
r
for the orifice.
6. The centre of an orifice is situated 60cm above the bottom of a vessel
containing water to a depth of 2.4m. Assuming C
V
=0.98. Estimate how far the
water in the jet issuing from the orifice in the side of the vessel will fall in moving
horizontally a distance of 1m from the Vena contracta.
7. A large tank has a circular sharp edged orifice 2.5cm diameter in a vertical
side at a depth 0.6m below the constant water level. The jet diameter at Vena
contract is found to be 2cm. The water is found to be 2cm . The water discharged
is collected in a vessel having the inside dimensions of 1.2m x 0.6m x 0.6m. The
time required to fill the vessel is 7minutes. Calculate C
d
, C
V
& C
C.
8. Oil of S=0.85 issues from a 5cm diameter orifice under a pressure of 12N/cm
2
.
The diameter of jet at Vena contracta is 4cm and the discharge is 20 lps. What is
the coefficient of velocity?
9. A rectangular orifice 0.9m wide, 1.2m deep is discharging water from a vessel.
The top edge of the orifice is 0.6m below the water surface in the vessel.
Calculate the discharge through the orifice if C
d
=0.6 and the percentage error if
the orifice is treated as a small orifice.
10. A closed tank contains kerosene S=0.8 to a depth of 3m. The top portion of
the tank contains air under a pressure of 27.5kpa. If a sharp edged circular orifice
of 3cm diameter C
d
=0.62 is provided at the bottom of the tank, estimate the
discharge through the orifice.
11. A standard mouthpiece 5cm in diameter discharges water. Determine the
maximum head under which the flow would take place without the occurrence of
separation at Vena contracta. The coefficient of discharge for the mouth piece is
0.8, separation occurs when the absolute pressure is 2.44m of water.
12. Find the discharge through an external cylindrical mouth piece of 10cm in
diameter flowing under a head of 3.5m. What is the pressure at Vena contracta?
Take C
C
=0.62.
13. Calculate the value of V - notch angel to discharge 400lps under a head of
0.6m, assuming C
d
=0.60.
14. During a test in a laboratory the water passing through a venturimeter is
made to flow over a 90
0
V notch. Diameter at inlet is 25cm an that at the throat is
10cm. The pressure difference is 0.34m when the head over the V-notch is
steady at 18.2cm. If C
d
for the venturimeter is 0.97, what is the coefficient of
discharge of the V-Notch?
15. Discharge varies in a rectangular channel from 0.0057 to 0.142 m
3
/s. At what
height should a 90
0
V-notch be placed above the bed of the channel, if the depth
should not exceed 1.2m, in the channel? What will be its depth for minimum
discharge?
16. A 90
0
V-notch is provided in a rectangular channel 1.5m wide in order to
measure the channel discharge the channel is designed to carry a maximum
discharge of 0.4m
3
/s. With a depth not exceeding 1.8m. Find the position of the
apex of the notch from the bed of the channel.
17. A sharp crested rectangular weir 1.5m long and 90cm high is installed in a
rectangular channel 1.5m wide. If the head on the weir is 30cm, find the
discharge
a)Neglecting velocity of approach
b)Considering velocity of approach
18.A cipolletti weir of 40cm bottom width is Installed in a channel 75cm wide &
45cm deep. If the head over the weir crest is 25cm, find the discharge over the
weir.
a)Neglecting vel. of approach
b)Taking vel. of approach.
19. A rectangular channel is 4.5m wide. Water flows at a depth of 1.2m at a
velocity of 90cm/s. A sharp crested weir is constructed across the channel and
the depth in the channel rises upto 1.75m. What should be the height of the weir?
20.Two 90
0
V notches & one cipolletti weir are to be used side by side to
measure a discharge of 0.85m
3
/s through a channel. If the head should not
exceed 30cm, what should be the dimensions of the weirs?
21. Find the discharge through a trapezoidal notch which is 1.2m wide at the top
and 0.5m at the bottom and is 40cm in height. The head of water on the notch is
30cm. C
d
for the rectangular portion is 0.62 while for triangular portion is 0.60.
22. A discharge of 1500m
3
/s is to pass over a rectangular weir. The weir is
divided into a number of openings each of span 7.5m. If the velocity of approach
is 3m/s, find the number of openings needed in order the head of water over the
crest is not to exceed 1.8m.
23. A rectangular channel 1.5m wide has a discharge of 200lps, which is
measured by a right angled V notch. Find the position of the apex of the notch
from the bed of the channel if the maximum depth of water is not to exceed 1m.
Take C
d
=0.62.
24
. During an experiment in a laboratory 0.9 litres of water is flowing per second
over a V-notch when the head is 5cm and 5 litres/sec, when the head is 10cm.
Determine the values of k & n in the equation Q=KH
n
.
25. A triangular weir has one side sloping at 45
0
and the other side Z(H):1V.
Calculate the values of Z which gives a discharge of 0.12m
3
/s under a head of
0.3m. C
d
=0.6
26. Water flows through a 90
0
V-notch with a free surface of water at a height of
10cm from the crest. The corresponding flow rate is 1000lpm. If the height above
the crest is increased by 50%, what would be the new flow rate?
27. A rectangular channel 6m wide carries a flow of 1.5m
3
/s. A rectangular sharp
crested weir is to be installed near the end of the channel to create a depth of
1m. Upstream of the weir, calculate the necessary weir height. Assume C
d
=0.62.
28. A suppressed weir having a crest length of 4m, discharges under a head of
40cm. The height of the weir crest above the channel bed is 1m. Find the rate of
discharge.
29. A rectangular sharp crested weir is required to discharge 645 m
3
/s of water
under a head of 1.2m. If the coefficient of discharge is 0.6 and the velocity of
approach near the weir is 1 m/s, find the length of the weir required.
30. A discharge of 1800m
3
/s is to pass over a rectangular weir which is divided
into a number of spans of 10m each. If the velocity of approach is 4m/s, find the
number of opening required such that the head over the crest does not exceed
2m.
31. A waste weir has to pass a flood discharge of 3m
3
/s. find the length of the
broad crested weir to suit the discharge such that the depth of water over the
weir crest will not be more than 42cm. Take C
d
=0.97.
32. A 3m wide rectangular irrigation canal carries water with a discharge of
6m
3
/s. what height of rectangular weir installed across the canal will raise the
water surface to a level of 2m above the floor?
33. A rectangular weir 0.75m high & 1.5m long is to be used for discharging
water from a tank under a head of 0.5m. Estimate the discharge a) when it is
used as suppressed weir. b) when it is used as a contracted weir. Take C
d
=0.61.
34. A trapezoidal sharp crested weir has a base width of 1.2m & side slopes of
1.5H to 1V. Calculate the discharge over the weir for a head of 35cm. Take
C
d
=0.62.
28. Calculate the bottom width and side slopes of a trapezoidal notch to
discharge 4m
3
/s at a head of 1.5m and 1.2m
3
/s at a head of 0.75. Assume
C
d
=0.63.35. A Cipolletti weir of crest length 0.6m discharges water under of
0.36m. Find the discharge over the weir if the channel is 0.8m wide & 0.5m deep.
Take C
d
=0.6.

You might also like